Re: EQUILÁTERO

2001-03-18 Por tôpico José Paulo Carneiro



Considere um triangulo ABC inscrito neste circulo, fixe o lado 
BC e varie o vertice A na circunferencia. A area do triangulo vai variar e vai 
ser maxima quando for maxima a distancia de A ao lado BC (concorda?). E isto 
ocorrerah quando AB = AC (concorda?). 
Ou seja, para cada triangulo ABC inscrito, existe um outro 
A'BC (isosceles em A') que tem area maior do que ele.
Veja agora se da para utilizar este raciocinio para concluir 
que o maximo ocorre quando o triangulo eh equilatero.
JP


-Mensagem Original- 
De: Alexandre 
F. Terezan 
Para: OBM 
Enviada em: 18 de Março de 2001 03:59
Assunto: EQUILÁTERO

Provar (com noçoes de 2o grau de preferência) que, dada 
uma circunferencia de raio R, o triangulo nela inscrito de maior área é o 
triângulo equilátero de ladoigual a R x (sqrt 3)

Obs: x = produto
 sqrt 3 = raiz 
quadrada de 3 


Re: Ajuda urgente: cálculo do volume de um tanque.

2001-03-02 Por tôpico José Paulo Carneiro

Pessoal.
Era exatamente este tipo de reflexoes que eu queria suscitar. So que, como
vieram do grande Ralph, ja estao na sua forma final. Colocacoes "to the
point".
JP



-Mensagem Original-
De: "Ralph Costa Teixeira" [EMAIL PROTECTED]
Para: [EMAIL PROTECTED]
Enviada em: Sexta-feira, 2 de Maro de 2001 00:25
Assunto: Re: Ajuda urgente: clculo do volume de um tanque.


 Oi, Jose Paulo.

 O seu ponto eh valido. Na minha opiniao, nenhum problema terrivele em
 ficar com um problema numerico nas maos. A integral eh feia para
 resolver "no braco", mas nem sempre uma formula "fechadinha" (no caso
 aqui, sem integrais, usando soh as funcoes mais "comuns" como
 exponenciais, trigonometricas e aritmeticas) eh melhor do que uma que
 tenha um sinal de integral. Afinal, mesmo que a resposta fosse V=e^h,
 o calculo disso acaba sendo feito "numericamente" na hora de marcar o
 volume lah na escala do tanque. E esse calculo tambem envolve um erro
 numerico, assim como o da integral --  e ambos os erros podem ser bem
 controlados (dependendo do que estah dentro da integral).

 Ha de se lembrar que muitas das funcoes que a gente aceita facilmente
 podem muito bem ser consideradas como apelidos para integrais... Por
 exemplo, muita gente DEFINE a funcao log natural por

 ln x = INT (t=1 a t=x) 1/t dt

 e entao DEFINE e^x como a funcao inversa de ln x. Assim, tais funcoes
 nao seriam "melhores" que expressoes integrais...

 Talvez esse outro exemplo reforce o meu ponto: a nem tao conhecida
 funcao erf(x) eh definida como:

 erf(x) = 2/sqrt(Pi) INT (t=0 a t=x) e^(-t^2) dt

 Muita gente diria que a integral do lado direito "nao tem solucao".
 Mas, se voce conhece erf(x), a integral eh trivial. Se voce trabalhar
 bastante com a erf(x) e acostumar-se com suas propriedades, voce acaba
 aceitando-a tanto como e^x ou ln(x) ou cos(x) -- diga-se de passagem,
 a funcao erf de fato tem varias propriedades interessantes e aparece
 naturalmente em varios contextos, especialmente quando falando da
 distribuicao normal de probabilidade; suas tabelas sao bem conhecidas,
 seu comportamento eh bem entendido. Hoje em dia, se eu me deparo com
 uma resposta que tenha a integral de e^(-t^2) na expressao, eu me dou
 por satisfeito e considero o problema resolvido, mesmo que eu nao
 chegue a escrever erf(x) no lugar da integral.

 Assim, nao vejo nada de intrinsicamente terrivel na integracao
 numerica nao... Mas vejo como a minha mensagem anterior parece indicar
 isso. :) Eu eh que fiquei meio decepcionado de nao conseguir uma
 formula "fechadinha" para o problema... :) :) Uma das principais
 razoes da minha decepcao eh que o Software que eu costumo usar para
 gerar graficos nao consegue lidar automaticamente com a funcao
 definida via a integral (ele se confunde e acha que ha variaveis
 demais na expressao), mas conseguiria se eu arrumasse a tal formula
 fechadinha (bom, ele conhece erf(x), mas nao conhece aquela integral
 do volume do tanque). Talvez o Matlab consiga lidar com as duas formas
 igualmente? Nao sei...

 Abraco,
 Ralph

 Jos Paulo Carneiro wrote:
 
  Proponho que se rediscuta o conceito de integral feia. Qual eh o
problema de
  calcular uma integral numericamente?
  (So para provocar...)
  JP
 





Re: Ajuda urgente: cálculo do volume de um tanque.

2001-03-02 Por tôpico José Paulo Carneiro

Pessoal.
Era exatamente este tipo de reflexoes que eu queria suscitar. So que, como
vieram do grande Ralph, ja estao na sua forma final. Colocacoes "to the
point".
Eu acrescentaria, para provocar mais a todos, o seguinte:
Desde Arquimedes (ou antes, de Eudoxo?) se calculam integrais, por metodos
relativamente proximos da definicao. O chamado "Teorema fundamental do
Calculo" (sec.XVII), que permite "calcular" certas integrais em "segundos",
sabendo de cor um primitiva (ou tendo uma tabela delas) eh sem duvida uma
grande invencao da humanidade.
Mas acho que o ensino do Calculo criou uma especie de expectativa de que
toda integral tem de ser calculada desta forma. Dahi a pletora dos
"metodos de integracao", para tentar fazer as mais complicadas se moldarem a
tabela (o que alias tambem gera problemas divertidos).
Fica ainda na cabeca das pessoas que resolver "numericamente" uma integral
eh uma especie de vergonha, uma capitulacao do Departamento do Calculo
diante do Departamento de Calculo Numerico (que tal fundir estes 2
departamentos num so?).
Na verdade, como o Ralph demonstrou com os seus exemplos, isto eh ilusorio.
Eh apenas uma questao de saber de onde se parte.
Acho que estah em tempo de o ensino de Calculo (pelo memos em cursos de
Matematica) apresentar a integral como: 1) a medida de um "estoque"; 2) um
excelente instrumento para justificar "existencia" em matematica (como no
caso do logaritmo, citado pelo Ralph). E nao como aquela cobrinha sem
sentido que eh "o contario da derivada".
JP



JP



-Mensagem Original-
De: "Ralph Costa Teixeira" [EMAIL PROTECTED]
Para: [EMAIL PROTECTED]
Enviada em: Sexta-feira, 2 de Maro de 2001 00:25
Assunto: Re: Ajuda urgente: clculo do volume de um tanque.


 Oi, Jose Paulo.

 O seu ponto eh valido. Na minha opiniao, nenhum problema terrivele em
 ficar com um problema numerico nas maos. A integral eh feia para
 resolver "no braco", mas nem sempre uma formula "fechadinha" (no caso
 aqui, sem integrais, usando soh as funcoes mais "comuns" como
 exponenciais, trigonometricas e aritmeticas) eh melhor do que uma que
 tenha um sinal de integral. Afinal, mesmo que a resposta fosse V=e^h,
 o calculo disso acaba sendo feito "numericamente" na hora de marcar o
 volume lah na escala do tanque. E esse calculo tambem envolve um erro
 numerico, assim como o da integral --  e ambos os erros podem ser bem
 controlados (dependendo do que estah dentro da integral).

 Ha de se lembrar que muitas das funcoes que a gente aceita facilmente
 podem muito bem ser consideradas como apelidos para integrais... Por
 exemplo, muita gente DEFINE a funcao log natural por

 ln x = INT (t=1 a t=x) 1/t dt

 e entao DEFINE e^x como a funcao inversa de ln x. Assim, tais funcoes
 nao seriam "melhores" que expressoes integrais...

 Talvez esse outro exemplo reforce o meu ponto: a nem tao conhecida
 funcao erf(x) eh definida como:

 erf(x) = 2/sqrt(Pi) INT (t=0 a t=x) e^(-t^2) dt

 Muita gente diria que a integral do lado direito "nao tem solucao".
 Mas, se voce conhece erf(x), a integral eh trivial. Se voce trabalhar
 bastante com a erf(x) e acostumar-se com suas propriedades, voce acaba
 aceitando-a tanto como e^x ou ln(x) ou cos(x) -- diga-se de passagem,
 a funcao erf de fato tem varias propriedades interessantes e aparece
 naturalmente em varios contextos, especialmente quando falando da
 distribuicao normal de probabilidade; suas tabelas sao bem conhecidas,
 seu comportamento eh bem entendido. Hoje em dia, se eu me deparo com
 uma resposta que tenha a integral de e^(-t^2) na expressao, eu me dou
 por satisfeito e considero o problema resolvido, mesmo que eu nao
 chegue a escrever erf(x) no lugar da integral.

 Assim, nao vejo nada de intrinsicamente terrivel na integracao
 numerica nao... Mas vejo como a minha mensagem anterior parece indicar
 isso. :) Eu eh que fiquei meio decepcionado de nao conseguir uma
 formula "fechadinha" para o problema... :) :) Uma das principais
 razoes da minha decepcao eh que o Software que eu costumo usar para
 gerar graficos nao consegue lidar automaticamente com a funcao
 definida via a integral (ele se confunde e acha que ha variaveis
 demais na expressao), mas conseguiria se eu arrumasse a tal formula
 fechadinha (bom, ele conhece erf(x), mas nao conhece aquela integral
 do volume do tanque). Talvez o Matlab consiga lidar com as duas formas
 igualmente? Nao sei...

 Abraco,
 Ralph

 Jos Paulo Carneiro wrote:
 
  Proponho que se rediscuta o conceito de integral feia. Qual eh o
problema de
  calcular uma integral numericamente?
  (So para provocar...)
  JP
 








Re: Ajuda urgente: cálculo do volume de um tanque.

2001-03-01 Por tôpico José Paulo Carneiro

Proponho que se rediscuta o conceito de integral feia. Qual eh o problema de
calcular uma integral numericamente?
(So para provocar...)
JP



-Mensagem Original-
De: "Ralph Costa Teixeira" [EMAIL PROTECTED]
Para: [EMAIL PROTECTED]
Enviada em: Quinta-feira, 1 de Maro de 2001 19:46
Assunto: Re: Ajuda urgente: clculo do volume de um tanque.



 Oi todo mundo.

 Voltando ao problema do tanque deitado, as noticias nao sao nada boas
 para o resto do problema. Acaba numa integral muito feia que eu creio
 soh poder ser feita mesmo numericamente.


 I. O CILINDRO

 Na ultima mensagem eu disse que, se o nivel do liquido eh h a partir do
 fundo de um cilindro de raio r e "comprimento" a (pois o cilindro estah
 deitado), entao o volume do liquido lah dentro eh:

 V1 = a.r^2.  [Pi + (m-1)sqrt(m(2-m)) - arccos(m-1)]

 onde eu uso m=h/r para facilitar as coisas. Seria legal marcar o zero
 da escala no centro do cilindro, isto eh, tomar h1 = h-r como variavel
 ao inves de h. Assim, se m=h1/r

 V1 = a.r^2. [Pi + m.sqrt(1-m^2) - arccos(m)]

 Daqui por diante vou usar esta notacao, marcando h=0 no meio, e assim h
 vai de -r a r. Quem nao gostar, troque h por h+r de volta. :)


 II. CADA UMA DAS CALOTAS

 Uma secao *horizontal* da calota esferica aa altura z (z=0 eh o plano
 horizontal passando pelo centro da esfera) eh um segmento circular. Eu
 peguei uma destas secoes HORIZONTAIS e desenhei-a aqui vista de cima,
 preenchida com s's. O x marca o centro do circulo, R0 eh seu raio e d eh
 a distancia entre o centro e o segmento que delimita o segmento
 circular.

  |\
  |s\
  |ss\
   d  |sss|
 x|sss|
  \   |sss|
   \  |ss/
  R0\ |s/
 \|/

 Como a secao horizontal estah aa distancia |z| do centro da esfera,
 temos R0=sqrt(R^2-z^2).

 Por outro lado, pode-se notar que d eh tambem a distancia do centro da
 ESFERA (que nao eh necessariamente x! O centro da esfera estah na secao
 horizontal z=0!) ao plano usado para corta-la em uma calota. Em outras
 palavras, d=sqrt(R^2-r^2).

 Enfim, lembre-se que a area do segmento circular eh a area de um setor
 circular menos um triangulo escolhidos a dedo... A formula eh:

 A = (R0)^2.arccos(d/R0) - d.sqrt(R0^2-d^2)

 Substitua R0 e d:

 A = (R^2-z^2).arccos(sqrt(R^2-r^2)/sqrt(R^2-z^2))
 - sqrt(R^2-r^2).sqrt(r^2-z^2)

 Agora voce teria que integrar isso de z=-r a z=h para achar o volume do
 liquido. A segunda parte da integral (a segunda linha da area) eh facil
 por substituicao, eh igual ao calculo feito para o cilindro. Tome de
 novo m=h/r e fique com:

 V3 = -r^2.sqrt(R^2-r^2). [Pi + m.sqrt(1-m^2) - arccos m]

 A primeira parte eh pior ainda. Use z=R.cost, r/R=p e h/R=q para obter:

 V2 = R^3 INT(t = arccos(q)  a   t = Pi - arccos(p))
  (sint)^3 . arccos(sqrt(1-p^2)/sint) dt

 Ateh onde eu sei, esta integral nao pode ser resolvida analiticamente
 (o arccos(K/sint) me faz acreditar nisto), a menos eh claro que p=1 (o
 caso em que r=R, ou seja, em que as calotas sao de fato dois
 hemisferios).

 Assim, a melhor opcao eh fazer um calculo numerico desta integral
 usando os seus dados a=14500, r=500 e R=3142... Note que V2 depende de q
 de maneira "simples". Ponha varios valores de q e faca uma tabela... :(


 III. JUNTANDO TUDO

 Em suma, pegue um computador e calcule as seguintes quantidades para
 cada h desejado de -r a r:

 p=r/R; q=h/R; m=h/r=q/p;

 DENTRO DO CILINDRO:
 V1 = a.r^2.[Pi + m.sqrt(1-m^2) - arccos m]

 NAS CALOTAS:
 2V2 = R^3 INT(t = arccos(q); t = Pi - arccos(p))
  (sint)^3 . arccos(sqrt(1-p^2)/sint) dt

 (Resolva numericamente para o valor p fixo que voce tem e usando
 diversos valores de q)

 2V3 = -r^2.sqrt(R^2-r^2). [Pi + m.sqrt(1-m^2) - arccos m]


 O volume que voce quer eh V1+2V2+2V3.

 Eu sei que a resposta parece um pouco decepcionante, mas espero que
 tenha ajudado. As vezes eh mais facil fazer ao contrario: vah enchendo o
 tanque com volumes conhecidos e marcando os valores de h para cada um,
 montando assim a escala "experimentalmente"... Ou faca isso para um
 tanque igual mas menor em escala... :)

-
  / \
 /   \
| |
 \---/
  \ /
   -
  
  
  Nesse caso temos:
  a = 14500mm; r = 500mm; R = 3142mm;
  em que, a = comprimento do cilindro (no considerar as calotas, e sim
apenas
  o cilindro plano nos lados); r = raio do cilindro; R = raio da calota
at a





Re: Problema com compasso

2001-03-01 Por tôpico José Paulo Carneiro

Wagner:
Quais sao as regras deste compasso?
Ele so pode tracar uma circunferencia com centro dado e passando por um
ponto dado?
Neste caso, tenho dificuldade com o passo "trace C5(D,2)", pois ainda nao
sei que os pontos D e ... (nao me lembro agora, pois nao tenho a figura na
feente) sao diametralmente opostos.
Por outro lado, se o compasso permite tracar um circulo de centro dado e
raio igual a distancia entre dois pontos dados, vou ter que construir
primeiro um ponto que diste 2 de um conhecido. Alem disto, vou ter um
problema no cabri, pois o "compasso" do Cabri exige um segmento previamente
tracado (regua...) ou entao o uso de uma medida numerica, que eh feio.
Nao sei se fui claro na minha duvida.
Abracos.
JP


-Mensagem Original-
De: "Eduardo Wagner" [EMAIL PROTECTED]
Para: [EMAIL PROTECTED]
Enviada em: Sexta-feira, 2 de Maro de 2001 06:39
Assunto: Re: Problema com compasso


 Nas construcoes com regua e compasso devemos perceber
 que o compasso eh, de certa forma, um instrumento mais
 nobre que a regua. Ele traca uma circunferencia perfeita
 enquanto que a regua depende da precisao de sua fabricacao.

 O Nicolau lembrou um belo teorema que afirma que qualquer
 construcao que pode ser realizada com regua e compasso
 pode ser tambem realizada apenas com o compasso.
 Este teorema se deve a um matematico dinamarques
 chamado Georg Mohr(1640-1697) e foi publicado em 1672,
 mas, aparentemente, ninguem deu importancia.
 Mais de um seculo depois, Lorenzo Mascheroni redescobriu
 esta perola e publicou um livro sobre construcoes geometricas
 apenas com o compasso e dai este tema passou a ser conhecido e
 apreciado.

 Vamos resolver o problema de encontrar o ponto medio
 de um segmento AB usando apenas o compasso.
 Na notacao que vamos utilizar, C(P,r) indica uma
 circunferencia de centro P e raio r, e C1 X C2
 indica um dos pontos de intersecao entre as circunferencias
 C1 e C2.

 Seja AB = 1.
 Trace C1(A,1)
 Trace C2(B,1)
 P = C1 X C2
 Trace C3(P,1)
 Q = C3 X C2
 Trace C4(Q,1)
 D = C4 X C2
 Trace C5(D,2)
 E,F = C5 X C1
 Trace C6(E,1)
 Trace C7(F,1)
 M = C6 X C7 eh o ponto medio de AB. Prove!

 Abraco,
 Wagner.

 --
 From: "Nicolau C. Saldanha" [EMAIL PROTECTED]
 To: [EMAIL PROTECTED]
 Subject: Re: Problema com compasso
 Date: Tue, Feb 27, 2001, 4:55
 

 
 
  On Mon, 26 Feb 2001, Bruno F. C. Leite wrote:
 
  Dado o segmento AB, ache o ponto mdio de AB, USANDO SOMENTE O
COMPASSO.
 
  Bruno Leite
 
 
  No vou resolver, mas quero chamar a ateno de que este problema
   caso particular do seguinte:
 
  Dados pontos A, B, C e D no plano encontre a interseo entre
  as retas AB e CD usando apenas compasso.
 
  O seu problema se reduz facilmente a este:
  basta traar um crculo de centro A e raio AB e outro
  de centro B e mesmo raio. Chame as duas intersees de C e D.
 
  Outro problema:
 
  Dados pontos A, B, C e D no plano encontre as intersees
  entre o crculo de centro A e raio AB e a reta CD,
  novamente usando apenas compasso.
 
  Dados estes dois problemas, no  difcil provar que
  *qualquer* construo que pode ser realizada com rgua e compasso
  pode ser tambm realizada apenas com compasso.
 
  []s, N.
 
 
 





Re: Godel

2001-02-01 Por tôpico José Paulo Carneiro

O livro Godel's Proof eh de Nagel e Newman
JP

-Mensagem original-
De: Rogerio Fajardo [EMAIL PROTECTED]
Para: [EMAIL PROTECTED] [EMAIL PROTECTED]
Data: Quinta-feira, 1 de Fevereiro de 2001 13:17
Assunto: Re: Godel


J foi citado um muito bom, aqui na lista.
http://www.dmm.im.ufrj.br/diversos/godel.htm
 um bom comeo. Apesar de ser um pouco informal,  timo para dar a idia
geral do teorema. Depois vc procura algum material com a demonstrao mais
formal. Para isso, no conheo nenhum site, mas vc pode ler o livro
"Godel's
Proof" ou o teorema original de Godel: "On formally undecidable
propositions
of Principia Mathematica and related systems", da editora Dover. Esse
ltimo
 muito tcnico, mas tem uma introduo que ajuda bastante a compreenso do
teorema.

Rogrio


From: "Bruno Woltzenlogel Paleo" [EMAIL PROTECTED]
Reply-To: [EMAIL PROTECTED]
To: "Olympium" [EMAIL PROTECTED],Cincia-List
[EMAIL PROTECTED],"OBMList" [EMAIL PROTECTED]
Subject: Godel
Date: Thu, 1 Feb 2001 08:55:57 -0200

Alguem conhece algum bom site sobre o teorema de Godel?

at mais...

Bruno Woltzenlogel Paleo
http://br.geocities.com/dopelganger5/
[EMAIL PROTECTED]


_
Get Your Private, Free E-mail from MSN Hotmail at http://www.hotmail.com.






Re: Um pouco de fisica

2001-01-18 Por tôpico José Paulo Carneiro

Lembro-me de existir uma apostila antiga do Elon Lages Lima: Elementos de
Calculo Tensorial. Ela apresenta nao a visao original do Calculo Tensorial,
vendo os tensores como agentes de mudanacas de coordenadas, mas a visao
"moderna" (falo de meio seculo atras), em termos de espacos (vetoriais)
duais. Nao sei se virou livro, mas de qualquer forma tem no IMPA.
JP
-Mensagem original-
De: Leonardo Motta [EMAIL PROTECTED]
Para: [EMAIL PROTECTED] [EMAIL PROTECTED]
Data: Quinta-feira, 18 de Janeiro de 2001 00:07
Assunto: Re: Um pouco de fisica


Talvez isso ajude:

Grosso modo um tensor eh uma grandeza vetorial e escalar, um grupo
algebrico
nao-linear que sofre transformacoes bem definidas. Uma matriz e' um caso
particular de tensor, por exemplo. O calculo tensorial tambem pode ser
visto
como a generalizacao da algebra linear, um objeto mais geral que o vetor e
a
matriz. A definicao formal poderia ser encontrada na MathWorld da
Treasure-Troves of Science, do Dr. Weisstein da WOLFRAM, se a enciclopedia
nao estivesse offline (temporariamente): http://www.treasure-troves.com/






Re: limite do M.H.S.

2001-01-16 Por tôpico José Paulo Carneiro



O limite procurado eh o limite de
[A.cos(w(t+h)+f) - A.cos(wt+f)] / h, quando h tende a 
0.
Agora aplique a formula cos(p)-cos(q)=2 
sen[(p+q)/2].sen[(q-p)/2], para tornar aquela expressao igual a:
-2A multiplicado por sen(wt+f+wh/2) [este fator tende a 
sen(wt+f)], multiplicado por
sen(wh/2)/h. Este ultimo fator eh o mesmo que w/2 vezes 
sen(u)/u, onde u=wh/2 estah tendendo a 0.
Por um resultadoclassico de limites, sen(u)/u tende a 1 
quando u tende a 0. Logo, o limite em questao eh:
-2A.w/2.sen(wt+f) = -2Aw.sen(wt+f)

Conferiu com o que voce achou geometricamete?
JP



-Mensagem original-De: 
Daniel [EMAIL PROTECTED]Para: 
Lista da OBM [EMAIL PROTECTED]Data: 
Quarta-feira, 17 de Janeiro de 2001 00:50Assunto: limite do 
M.H.S.
  
 Por um acaso poderiam me ajudar com um limite, matéria da 
qual ainda não estudei tudo? É o seguinte:
  
 No Movimento Hamônico Simples, a função horária de elongação 
é dada por:

  x = 
A.cos(wt+f), 

 Consegui 
deduzir a função da velocidade usando trigonometria, mas sei que

  v = lim 
Dx/Dt, quando Dt tende a zero,
 A pergunta é como calcular tal 
limite da função horária acima?

  

   
Daniel


Re: Polinômio unitário

2001-01-15 Por tôpico José Paulo Carneiro



Pode ser que esteja querendo dizer que o coeficiente do termo 
de mais alto grau eh 1.
Em geral se chama isto de monico.

Ou entao que tal polinomio eh uma "unidade", termo usado por 
alguns em um dominio de integridade (ou seja, um anel sem divisores de zero) 
para dizer que o polinomio eh invertivel. Por exemplo, se os coeficientes do 
polinomio estao em um corpo (por exemplo, R ou Q), os invertiveis sao os 
polinomios de grau zero.
JP

-Mensagem original-De: 
Daniel [EMAIL PROTECTED]Para: 
Lista da OBM [EMAIL PROTECTED]Data: 
Terça-feira, 16 de Janeiro de 2001 00:15Assunto: Polinômio 
unitário
  
 Por um acaso alguém sabe o que um autor quer dizer quando 
fala que um polinômio é unitário?

  


Daniel


Re: Questao legal!

2001-01-10 Por tôpico José Paulo Carneiro

Exatamente.
Idealmente, aumentando o numero de operacoes, a precisao deveria aumentar.
Mas cada raiz quadrada (por exemplo) feita na calculadora (ainda mais a do
feirante) introduz
um erro de arredondamento / truncamento, que vai se acumulando. Neste
sentido, o numero de operacoes joga (no sentido da "teoria dos jogos")
contra a precisao.
Volto a citar o meu artigo (Nota) do Math.Gazette, onde este ponto eh
comentado.
Que haja um ponto "otimo", foi observado empiricamente (e este otimo varia
ligeiramente com o proprio valor do numero cujo logaritmo se estah
biscando), mas nao justificado matematicamente. Se me lembro bem, o
"referee" acrescentou uma frase do tipo: "seria um interessante assunto de
pesquisa pesquisar este otimo"...
JP



-Mensagem original-
De: Nicolau C. Saldanha [EMAIL PROTECTED]
Para: [EMAIL PROTECTED] [EMAIL PROTECTED]
Data: Tera-feira, 9 de Janeiro de 2001 11:26
Assunto: Re: Questao legal!




On Mon, 8 Jan 2001, Jos Paulo Carneiro wrote:

 No caso que eu propus, nao ha funcoes trigonometricas.
 Trata-se da calcladora "do feirante".
 So tem as 4 operacoes e mais raiz quadrada.

 Atencao, problemistas!
 Como sei que voces ja pensaram, aqui vai:
 Tome uma calculadora do feirante e teclem o numero dado positivo N.
 Agora aperte a tecla da raiz quadrada 15 vezes.
 Subtraia 1 do resultado.
 Agora multiplique o resultado por 2. De novo multiplique por 2, ateh
 completar
 15 multiplicacoes por 2.
 O resultado eh (uma aproximacao de) logaritmo natural de N.

 Observacoes:
 1) Nao ha nada de magico com o numero 15. Voce pode fazer com qualquer
 numero de vezes em vez de 15 (desde que o numero de raizes quadradas seja
o
 mesmo que o de multiplicacoes por 2), obtendo aproximacoes com precisoes
 diferentes.

[...]

Para quem tiver pacincia, vale a pena fazer o teste.
Variando este nmero N de vezes que apertamos a tecla [sqrt],
comeando com valores baixos de N e subindo,
a preciso vai aumentando, mas s at um certo ponto.
Depois de um certo valor de N a preciso comea a piorar.

As duas perguntas so:
(a) qual a razo de ser deste estranho comportamento?
(b) qual o valor ideal de N? O que faz dele o valor ideal?

[]s, N.






Re: Integral

2001-01-07 Por tôpico José Paulo Carneiro



Um bonito exercicio eh mostrar que a integral de 0 a infinito 
de sen(t)/t eh igual pi/2.
Mais bonita ainda eh a demonstracao usando integracao no plano 
complexo.
JP

-Mensagem original-De: 
Eric Campos Bastos Guedes [EMAIL PROTECTED]Para: [EMAIL PROTECTED] [EMAIL PROTECTED]Data: 
Domingo, 7 de Janeiro de 2001 11:56Assunto: RES: 
Integral

   Alguém pode me ajudar com essa integral 
  ?
  
   
  
  Tentei, várias vezes seguidas, usar o método de integração 
  por partes, mas sempre "zerava" ...
  
   Davidson
  
  Acho 
  que você não vai conseguir expressar essa integral com funções elementares, 
  mas parece que consegui uma série que converge para a tal integral, partindo 
  da expressão para sen(t) em série, que é:
  
  sen(t) = t - t^3/3! + t^5/5! - t^7/7! + 
  t^9/9! - ..., logo
  
  sen(t)/t = 1 - t^2/3! + t^4/5! - t^6/7! + t^8/9! - 
  ... , integrando vem:
  
  integral(sen(t)/t) = t - t^3/(3*3!) + 
  t^5/(5*5!) - t^7/(7*7!) + t^9/(9*9!) - ...
  
  Eric.


Re: Questao legal!

2001-01-05 Por tôpico José Paulo Carneiro

Vou acrescentar outro problema:
Agora, a calculadora tem as 4 operacoes e mais a tecla da raiz quadrada
(como alias eh o caso das calculadoras mais simples que existem).
Explique como se pode calcular o logaritmo natural de um numero positivo.

[Se voce quiser a solucao, pode encontra-la na minha "Nota" :
Logarithms on the Simplest Calculator – The Mathematical Gazette
– Vol. 82; Number 493; March 1998.
Esta revista inglesa pode ser consultada,
por exemplo, na Biblioteca do IMPA]
JP



-Mensagem original-
De: Fabricio Damasceno [EMAIL PROTECTED]
Para: [EMAIL PROTECTED] [EMAIL PROTECTED]
Data: Quinta-feira, 4 de Janeiro de 2001 18:22
Assunto: Questao legal!


 Ola Pessoal!
 Recentemente encontrei um problema muito interessante.
"dispondo de uma calculadora simples que somente realiza as operacoes
fundamentais (+, -, *, /) e calcula as funcoes trigonometricas. Calcule
a raiz quadrada de um numero natural."


MailBR - O e-mail do Brasil -- http://www.mailbr.com.br
Faça já o seu. É gratuito!!!





Re: Questao legal!

2001-01-05 Por tôpico José Paulo Carneiro

Eh evidente que se trata de uma resposta aproximada.
No caso, trata-se de apertar certas teclas um certo numero n de vezes,
e o grau de aproximacao vai depender de n.
Alias, existe algum numero irracional que nao seja calculado por
aproximacao?
E mesmo um racional: calcule 1 dividido por 3 na calculadora, e
a resposta ja eh aproximada.
Como dizia o velho Dieudonne: calcular um numero real eh majorar, minorar,
aproximar.
JP

-Mensagem original-
De: Nicolau C. Saldanha [EMAIL PROTECTED]
Para: [EMAIL PROTECTED] [EMAIL PROTECTED]
Data: Sexta-feira, 5 de Janeiro de 2001 12:35
Assunto: Re: Questao legal!




On Fri, 5 Jan 2001, José Paulo Carneiro wrote:

 Vou acrescentar outro problema:
 Agora, a calculadora tem as 4 operacoes e mais a tecla da raiz quadrada
 (como alias eh o caso das calculadoras mais simples que existem).
 Explique como se pode calcular o logaritmo natural de um numero positivo.

 [Se voce quiser a solucao, pode encontra-la na minha "Nota" :
 Logarithms on the Simplest Calculator – The Mathematical Gazette
 – Vol. 82; Number 493; March 1998.
 Esta revista inglesa pode ser consultada,
 por exemplo, na Biblioteca do IMPA]
 JP

  Ola Pessoal!
  Recentemente encontrei um problema muito interessante.
 "dispondo de uma calculadora simples que somente realiza as operacoes
 fundamentais (+, -, *, /) e calcula as funcoes trigonometricas. Calcule
 a raiz quadrada de um numero natural."

Nestas questões acho que ajuda dar um enunciado mais completo.
Acho que queremos um algoritmo para calcular de forma rápida e simples
uma resposta aproximada, certo? No problema do JP sei que é impossível
obter uma resposta exata em um número finito de passos mas no segundo,
dado que podemos calcular funções trigonométricas, não tenho certeza.
De qualquer forma seria estranho falar de calculadora e esperar resposta
exata... []s, N.






Re: Publicação em primeiramão da constante de Hinrichs

2000-12-29 Por tôpico José Paulo Carneiro

Nao sei bem por que voce quer fazer isto "sem o computador", a nao ser que
voce queira dizer: "sem usar uma caixa preta, do tipo: computador, resolva a
equacao!", isto eh, suponho que voce queira entender o que um programa
desses faz.
Ha toda uma teoria muito bonita (e nao dificil) sobre as solucoes de
equacoes do tipo f(x)=x, ou seja, os pontos fixos da funcao f. A teoria
consiste em estabelecer as condicoes sob as quais a sequencia x(n+1)=f(x(n))
[os paretnteses sao de funcao, nao de multiplicacao] converge a um ponto
fixo, partindo de um valor inicial x(0).
Uma condicao suficiente para a convergencia eh que exista um intervalo J e
uma constante 0k1 tal que o modulo da derivada de f permaneca menor que k
nesse intervalo (o x(0) tambem deve ser tomado neste intervalo).
Os livros de Calculo Numerico tem isto.
JP


-Mensagem original-
De: Rodrigo Villard Milet [EMAIL PROTECTED]
Para: [EMAIL PROTECTED] [EMAIL PROTECTED]
Data: Sexta-feira, 29 de Dezembro de 2000 03:38
Assunto: Re: Publicação em primeiramão da constante de Hinrichs


Como faço para, sem o uso de computador, ter uma boa aproximação da raiz
dessa equação ???
Abraços,
¡ Villard !
-Mensagem original-
De: José Paulo Carneiro [EMAIL PROTECTED]
Para: [EMAIL PROTECTED] [EMAIL PROTECTED]
Data: Quinta-feira, 28 de Dezembro de 2000 21:03
Assunto: Re: Publicação em primeiramão da constante de Hinrichs


Esta constante eh a solucao da equacao cos(x)=x.
Voce pode fazer o mesmo com varias outras funcoes.
Por exemplo, com o seno vai dar 0.
Para outras funcoes, o processo nao vai convergir.
As vezes, vai convergir ou nao dependendo do ponto de partida.
Eu sempre usei este exemplo e outros semelhantes em aulas de Analise ou de
Calculo numerico, para introduzir processos iterativos, sitemas dinamicos,
etc.
JP
-Mensagem original-
De: Benjamin Hinrichs [EMAIL PROTECTED]
Para: [EMAIL PROTECTED] [EMAIL PROTECTED]
Data: Quinta-feira, 28 de Dezembro de 2000 13:09
Assunto: Publicação em primeiramão da constante de Hinrichs


Porto Alegre, 28 de Dezembro de 2000.

Srs.,
tenho o prazer de vos anunciar a novíssima constante de Hinrichs (não
reparem no meu sobrenome). É algo trivial, pegue uma calculadora não
muito pomposa, ligue, aperte cos e, com o resultado, tire o cosseno e
novamente e assim em diante. Vc está se aproximando da constante de
Hinrichs. O princípio é trivial, é claro.
Não pergunte a utilidade pois alguns dos trabalhos de Riemann só vieram a
ser usados 50 anos depois por Einstein.

Espero que ninguém conheço outro nome para a constante de Hinrichs.

Aliás, falando de Einstein, quem me diz qual é a relação entre E=mc^2 e a
teoria da relatividade, afinal a fórmula é frequentemente associada à
teoria da relatividade.

Grande abraço,

Benjamin Hinrichs











Re: Publicação em primeiramão da constante de Hinrichs

2000-12-28 Por tôpico José Paulo Carneiro

Esta constante eh a solucao da equacao cos(x)=x.
Voce pode fazer o mesmo com varias outras funcoes.
Por exemplo, com o seno vai dar 0.
Para outras funcoes, o processo nao vai convergir.
As vezes, vai convergir ou nao dependendo do ponto de partida.
Eu sempre usei este exemplo e outros semelhantes em aulas de Analise ou de
Calculo numerico, para introduzir processos iterativos, sitemas dinamicos,
etc.
JP
-Mensagem original-
De: Benjamin Hinrichs [EMAIL PROTECTED]
Para: [EMAIL PROTECTED] [EMAIL PROTECTED]
Data: Quinta-feira, 28 de Dezembro de 2000 13:09
Assunto: Publicação em primeiramão da constante de Hinrichs


Porto Alegre, 28 de Dezembro de 2000.

Srs.,
tenho o prazer de vos anunciar a novíssima constante de Hinrichs (não
reparem no meu sobrenome). É algo trivial, pegue uma calculadora não
muito pomposa, ligue, aperte cos e, com o resultado, tire o cosseno e
novamente e assim em diante. Vc está se aproximando da constante de
Hinrichs. O princípio é trivial, é claro.
Não pergunte a utilidade pois alguns dos trabalhos de Riemann só vieram a
ser usados 50 anos depois por Einstein.

Espero que ninguém conheço outro nome para a constante de Hinrichs.

Aliás, falando de Einstein, quem me diz qual é a relação entre E=mc^2 e a
teoria da relatividade, afinal a fórmula é frequentemente associada à
teoria da relatividade.

Grande abraço,

Benjamin Hinrichs







Re: Pode-se ordenar um conjunto numeroso?

2000-12-21 Por tôpico José Paulo Carneiro

O conjunto dos  complexos pode ser ordenado (de infinitas maneiras).
O que nao se pode eh dota-lo de uma ordem que seja compativel com as suas
operacoes de corpo (ou seja, ele eh um corpo nao-ordenavel), no sentido de
que:
ab implique a+cb+c e ab e c0 implique acbc.
JP

-Mensagem original-
De: Jorge Peixoto Morais [EMAIL PROTECTED]
Para: [EMAIL PROTECTED] [EMAIL PROTECTED]
Data: Quinta-feira, 21 de Dezembro de 2000 04:33
Assunto: Pode-se ordenar um conjunto numeroso?


Um conjunto de alta cardinalidade, como 2^(cardinalidade de R), pode ser
ordenado (como R, apesar de que C tem a mesma cardinalidade e nao pode)?
Existe algum conjunto nessas condicoes?

A uniao e o produto cartesiano de uma familia enumeravel de conjuntos cuja
cardinalidade eh a de R teem que cardinalidade? E se, em vez de uma familia
enumeravel, tem-se (por exemplo) uma bijecao entre cada conjunto e o
conjunto dos reais?

[]s, J.P.
_
Get Your Private, Free E-mail from MSN Hotmail at http://www.hotmail.com.






Re: Integral

2000-12-07 Por tôpico José Paulo Carneiro

Em primeiro lugar, eh preciso estar claro a que integral voce se refere.
Se a integral em questao eh a de Riemann, em um intervalo da forma [a;b],
entao a continuidade eh suficiente (esta continuidade eh tambem suficiente
para garantir a existencia de uma primitiva), mas nao necessaria: basta
pensar em uma funcao que eh zero para x entre 0 e 1, e 1 nos extremos 0 e 1.
Esta funcao eh Riemann-integravel, sua integral eh zero e ela eh discontinua
em 2 pontos do intervalo.
Uma  condicao necessaria e suficiente eh que ela seja limitada e que o
conjunto de seus pontos de descontinuidade tenha comprimento (medida de
Lebesgue) igual a zero.
JP

-Mensagem original-
De: Leonardo Motta [EMAIL PROTECTED]
Para: [EMAIL PROTECTED] [EMAIL PROTECTED]
Data: Quinta-feira, 7 de Dezembro de 2000 22:36
Assunto: Re: Integral


 Quais são os critérios, para sabermos se uma função é integrável?

A funcao deve ser continua no intervalo em questao e deve-se conhecer uma
funcao derivada identica a funcao que se deseja integrar... Creio que soh!
:)







Re: Fofoca Matematica.

2000-12-06 Por tôpico José Paulo Carneiro

Super-parabens para os tres brasileiros!
JP

-Mensagem original-
De: Olimpiada Brasileira de Matematica [EMAIL PROTECTED]
Para: [EMAIL PROTECTED] [EMAIL PROTECTED]
Data: Quarta-feira, 6 de Dezembro de 2000 12:39
Assunto: Fofoca Matematica.





24 horas de competicao sem parar no Mat-Quiz!! 

No passado dia 4 de dezembro `as 10:00 horas da manha 
deu-se inicio a uma curiosa competicao  para matematicos
(profissionais e alunos de doutorado) organizada pelo CRM
(Centre de Recerca Matematica)- Barcelona em comemoracao 
ao ano Internacional da Matematica. 
O jogo foi disputado por equipes (sem limite no numero 
de integrantes) que competiram durante 24 horas seguidas 
via internet contra outros times do mundo todo. 
Para defender o Brasil inscreveram-se na brincadeira 3 
Matematicos: 
Krerley Oliveira, Fabio Brochero e Gugu estes "valentes" 
Matematicos enfrentaram o cansaco trancados na Biblioteca 
do IMPA competindo via internet contra mais de 100 times. 
A competicao acabou as 10 horas da manha de ontem e os caras
ficaram em terceiro e ganharam um computador da SUN.


Parabens!! (Haja folego hein,z!).

   
Abracos, 
Nelly.







Re: Hessiano

2000-12-04 Por tôpico José Paulo Carneiro



No lugar ij da matriz, estao as derivadas parciais do tipo df 
/ dx(i) dx(j).
O criterio de positividade eho mesmo que para qualquer matriz 
definida positiva:
todos os determinantes menores principais tem de ser 
positivos.
JP


-Mensagem original-De: 
Humberto Ferreira Vinhais [EMAIL PROTECTED]Para: 
Olimpíada de Matemática [EMAIL PROTECTED]Data: 
Segunda-feira, 4 de Dezembro de 2000 21:34Assunto: 
Hessiano
Hei, por favor, alguém poderia me responder se existe hessiano 
de ordem 3 ( para funções de 3 variáveis ) ? e se existe, como é que se escreve 
? e se através dele eu posso determinar pontos de máximo, mínimo e de sela de 
uma função da mesma forma que se determina isso para funções de 2 variáveis (, 
ou seja: H0 (ou del f / del x^2  0 =ponto de mínimo ou del f / del 
x^2 0 =ponto de máximo), H0 = ponto de sela e H=0 = inconclusivo 
)


Re: Ajudem-me com este polinomio.

2000-12-03 Por tôpico José Paulo Carneiro

O raciocinio estah perfeito, mas houve um erro de conta no delta, que eh:
4(2-a).

-Mensagem original-
De: Rodrigo Villard Milet [EMAIL PROTECTED]
Para: [EMAIL PROTECTED] [EMAIL PROTECTED]
Data: Segunda-feira, 4 de Dezembro de 2000 00:12
Assunto: Re: Ajudem-me com este polinomio.


Vemos que 1 e -1 são raízes de p(x). Daí, este é divisível por (x^2-1).
Fazendo a divisão pelo algoritmo da divisão, temos o seguite quociente :
 x^4 + 2x^3 + (a+1)x^2 + 2x + 1
Para acharmos suas raízes, devemos igualá-lo a zero, o q nos dá uma equação
recíproca !! . x^2 [ (x^2 +1/x^2) + 2(x + 1/x) + a + 1 ] = 0
...
 fazendo x+1/x = t , temos x^2 + 1/x^2 = t^2 - 2, daí a equação se
transforma em ...
 x^2 ( t^2 -2t + a - 1 ) = 0
Como queremos raízes reais, o delta de t^2 - 2t +a + 1 = 0 deve ser maior
ou
igual a zero. delta = 4 - 4(a+1) = - 4a. Daí, temos a=0 (I)
 As raízes são t = 1 +- sqrt(-a)
E, t = x + 1/x.
 (i) x + 1/x = 1 + sqrt(-a)
x^2 - (1 + sqrt(-a) )x +1 = 0. delta = 0 1 - a +
2sqrt(-a) - 4 = 0, . 2sqrt(-a) = 3 + a. Para -3 = a =
0,
temos -4a = 9 + 6a + a^2... a^2+10a+9=0 ...  -9= a = -1, o que
resulta em -3 = a = -1 (II)   ... Para a = - 3,
temos
a^2 + 10a + 9 = 0, o que nos dá apenas dois intervalos a = -9 e a = -1,
q
resulta em a = -9 (III) 

(ii) x +1/x = 1 - sqrt(-a)
x^2 - ( 1- sqrt(-a) )x +1 = 0. delta = zero, daí, temos : 1 -
2
sqrt(-a) -a -4=0 ...   2sqrt(-a) = -a -3, o q é absurdo, pois sqrt(-a) =
0

Daí, unindo (I), (II) e (III) , temos a = -9 ou -3= a =0.
Se não me engano, esta questão caiu no ITA em 97/98, e ñ sei se tinha essa
opção. Devo ter errado alguma conta confiram !

   Abraços,
  ¡Villard !



-Mensagem original-
De: Fabricio Damasceno [EMAIL PROTECTED]
Para: [EMAIL PROTECTED] [EMAIL PROTECTED]
Data: Domingo, 3 de Dezembro de 2000 21:05
Assunto: Ajudem-me com este polinomio.


 Seja "a" um numero real tal que o polinomio
p(x)= x^6 + 2x^5 + ax^4 - ax^2 - 2x -1 admite apenas raizes reais. Qual
o intervalo real ao qual "a" pertence?
MailBR - O e-mail do Brasil -- http://www.mailbr.com.br
Faça já o seu. É gratuito!!!







Re: questao do ITA

2000-12-01 Por tôpico José Paulo Carneiro

Este "B eh sempreinversivel" eh absurdo.
B nem eh quadrada.
JP

-Mensagem original-
De: Eduardo Quintas da Silva [EMAIL PROTECTED]
Para: [EMAIL PROTECTED] [EMAIL PROTECTED]
Data: Sexta-feira, 1 de Dezembro de 2000 12:14
Assunto: questao do ITA


Dizemos que duas matrizes n x m, A e B sao semelhantes se existe uma
matriz n x n inversível P tal que B = (P^-1).A.P. Se A e B sao matrizes
semelhantes quaisquer, entao

a) B e sempre inversivel
b) Se A e simetrica, entao B tambem e simetrica
c) B^2 e semelhante a A
d) Se C e semelhante a A, entao BC e semelhante a A^2
e) det(kI - B) = det(kI - A), onde k = numero real qualquer

P^-1 = matriz inversa de P
A^2 = A.A






Re: Radioatividade

2000-11-20 Por tôpico José Paulo Carneiro



Nao sei se eh bem isto que voce quer saber, mas o modelo 
matematico do decaimento radiativo postula que a taxa instantanea de variacao 
(perda) de massa de um elemento radiativo eh proporcional a massa existente no 
instante, ou seja : dm/dt = - km, onde k eh uma constante que depende da 
substancia em causa. 
A solucao desta equacao eh a funcao m(t)=m(0) 
exp(-kt).
A meia-vida eh o tempo necessario para que a substancia perda 
metade de sua massa, ou seja: fazendo m(t)=m(0)/2, encontra-se: t=(ln 
2)/k.
O interesse deste conceito eh justamente dar uma interpretacao 
concreta a constante k.
JP

-Mensagem original-De: 
Hugo Iver Vasconcelos Goncalves [EMAIL PROTECTED]Para: [EMAIL PROTECTED] [EMAIL PROTECTED]Data: 
Segunda-feira, 20 de Novembro de 2000 15:28Assunto: 
Radioatividade
Eu sei que isso nao tem muito a ver com a lista, mas será que 
alguém pode me enviar, ou dizer uma página ou livro onde eu possa encontrar a 
dedução da fórmula do periodo da meia-vida de um elemento raidioativo 



Re: Integral

2000-11-19 Por tôpico José Paulo Carneiro

Toda funcao continua eh (Riemann) integravel em qualquer intervalo fechado
limitado (em particular, f(x)=x^x=exp(x lnx) ). E mais: tem uma primitiva.
Outra questao diferente eh saber se esta primitiva tem uma expressao
simpatica em termos de um catalogo de funcoes "usuais", tais como
polinomios, quocientes de polinomios, funcoes trigonometricas, etc.
Mais uma vez, nao pensar que "integrar" uma funcao eh achar uma Primitiva
"camarada".  Muitas vezes, isto nem eh necessario. Eh o que se passa,
analogamente, com equacoes polinomiais que podem ser resolvidas por meio de
radicais e outras que so sao resolvidas por metodos numericos.
JP


-Mensagem original-
De: Leonardo Motta [EMAIL PROTECTED]
Para: [EMAIL PROTECTED] [EMAIL PROTECTED]
Data: Quinta-feira, 16 de Novembro de 2000 23:33
Assunto: Re: Integral


Escrevo para reforcar meu interesse nessa questao, a solucao da integral :
x^x. Esta nao e' uma forma padrao, e nao parece possibilitar muita
transformacao!!

Se alguem tiver a certeza de que essa integral nao tem solucao, por favor
dê
um toque! :)

[]'s
- Leonardo






Re: Questões_de_Trigonometria

2000-11-19 Por tôpico José Paulo Carneiro

sen x + cos x nao pode superar o seu proprio valor absoluto.
Por sua vez, estudar o maximo do modulo de sen x + cos x eh
o mesmo que estudar o maximo de seu quadrado, que eh
sen^2 x + cos^x + 2 sen x cos x = 1 + sen 2x, que atinge o valor
maximo 2, quando sen 2x = 1, isto eh, quando 2x = pi/2 + 2k pi,
ou seja,  x= pi/4 + k pi.
Para k par,  sen x+ cos x assume o valor maximo raiz de 2,
e para k impar, sen x + cos x assumeseu valor minimo menos raiz de 2.
JP



-Mensagem original-
De: Biscoito [EMAIL PROTECTED]
Para: [EMAIL PROTECTED] [EMAIL PROTECTED]
Data: Domingo, 19 de Novembro de 2000 19:06
Assunto: Re: Questões_de_Trigonometria


 Essa é a mais importante: qual o valor máximo de
 SenX + CosX ?

Como na relação de lançamento oblíquo, onde a
distância máxima possível no lançamento de qualquer
objeto é com o ângulo da metade do ângulo onde vc
obtém a altura máxima (90 graus), ou seja, 45 graus.
Da mesma forma q na física, 45 graus (pi/4) ou 135
graus (3*pi/4) representa o valor máximo desta soma,
creio q raiz de 2 (sqr2/2 + sqr2/2).

Vik



=
"Meu Deus, protegei-me de meus amigos!
Dos meus inimigos eu me encarregarei."

  Voltaire

__
Do You Yahoo!?
Yahoo! Calendar - Get organized for the holidays!
http://calendar.yahoo.com/





Re: trigonometria

2000-11-19 Por tôpico José Paulo Carneiro



Sugestao:
Sendo z = cos x + i senx, calcule a parte real de 
z+z^2+...+z^n (P.G.).
JP

-Mensagem original-De: 
filho [EMAIL PROTECTED]Para: 
discussão de problemas [EMAIL PROTECTED]Data: 
Domingo, 19 de Novembro de 2000 21:30Assunto: 
trigonometria
Demonstre que:

1/2 + cos x + cos 2x + cos 3x + 
... + cos nx = sen [( n + 1 / 2 )x] / [2. sen ( x 
/ 2 )]

para x diferente de k. 2 pi, k 
inteiro.


Re: Lógica?!

2000-11-18 Por tôpico José Paulo Carneiro

Estive fora, por isto estou respondendo atrasado.
So uma coisa: ninguem deve ter vergonha de perguntar nada.
Aposto que tem muita gente que fica sem saber, por nao perguntar.
JP

-Mensagem original-
De: Thomas de Rossi [EMAIL PROTECTED]
Para: [EMAIL PROTECTED] [EMAIL PROTECTED]
Data: Segunda-feira, 13 de Novembro de 2000 10:00
Assunto: Re: Lógica?!



Sds,

Por Favor, um pouco de paciência, mas vocês poderiam me explicar melhor o
"principio de Dirichlet", fico até com vergonha  "parece básica a questão",
mas eu não sei aplicar isso. E mesmo em que conteúdo se insere está
questão...
Alguém poderia me ajudar,

M.Obrigado, Thomas.

From: "Rodrigo Villard Milet" [EMAIL PROTECTED]
Reply-To: [EMAIL PROTECTED]
To: [EMAIL PROTECTED]
Subject: Re: Lógica?!
Date: Mon, 13 Nov 2000 01:32:55 -0200

As alternativas a,b,d estão colocadas fora de um contexto... portanto,
nada
pode se afirmar. Na letra e, vemos sua falsidade, pois por exemplo, todas
as
pessoas poderiam fazer aniversário em dezembro. No entanto, na letra c,
como
temos mais de 12 pessoas, vemos claramente ( pelo principio de dirichlet )
q
ela é verdadeira !
  Abraços,
 ¡ Villard !
-Mensagem original-
De: Thomas de Rossi [EMAIL PROTECTED]
Para: [EMAIL PROTECTED] [EMAIL PROTECTED]
Data: Segunda-feira, 13 de Novembro de 2000 00:43
Assunto: Lógica?!


 Pessoal,
 
 olhem só a questão abaixo...
 
 3) Em uma reunião, encontram-se 15 pessoas. Podemos afirmar que,
 necessariamente,
 
 (a) pelo menos uma delas tem mais de 30 anos.
 (b) pelo menos duas delas são do sexo feminino.
 (c) pelo menos duas delas fazem aniversário no mesmo mês.
 (d) pelo menos uma delas é brasileira.
 (e) pelo menos uma delas nasceu em Janeiro ou Fevereiro.
 
 Comentário: A questão abaixo foi colocada numa prova  de vestibular, eu
não
 sei se a escolha da alternativa é mais uma questão de lógica (sentimento
de
 chute), ou se tem como determinar matematicamente qual a resposta
correta?
 
 Qualquer ajuda será bem vinda,
 
 M.Obrigado,
 Thomas.
 

_
 Get Your Private, Free E-mail from MSN Hotmail at
http://www.hotmail.com.
 
 Share information about yourself, create your own public profile at
 http://profiles.msn.com.
 
 


_
Get Your Private, Free E-mail from MSN Hotmail at http://www.hotmail.com.

Share information about yourself, create your own public profile at
http://profiles.msn.com.






Re: Filosofia da Matemática

2000-11-18 Por tôpico José Paulo Carneiro

Indico o livro "What is Mathematics, Really?", de R.Hersch.
JP

-Mensagem original-
De: Rogerio Fajardo [EMAIL PROTECTED]
Para: [EMAIL PROTECTED] [EMAIL PROTECTED]
Data: Terça-feira, 14 de Novembro de 2000 10:24
Assunto: Filosofia da Matemática


Alguém pode me explicar o que são as correntes filosóficas da
Matemática: o Intucionalismo, o Formalismo e o Construtivismo (ou outras se
houver)?

   Grato,
   Rogério




_
Get Your Private, Free E-mail from MSN Hotmail at http://www.hotmail.com.

Share information about yourself, create your own public profile at
http://profiles.msn.com.






Re: Dúvida sobre Resíduos

2000-11-08 Por tôpico José Paulo Carneiro

Ola Marcos.
Existem dois livros editados pela SBM (SociedadeBrasileira de Matematica),
que nao custam caro (de 20,00 a 25,00, creio) e que tem muita coisa sobre
residuos, alem de outras interessantes. Um eh Introducao a Teoria dos
Numeros, de Jose Plinio de O.Santos, e outro (de leitura particularmente
agradavel) eh o Numeros Inteiros e Criptografia RSA, de Severino Collier
Coutinho.
Um sistema completo de residuos modulo m eh o conjunto dos possiveis restos
da divisao por m, ou entao qualquer outro conjunto de inteiros em que cada
um eh congruente a um desses restos. Por exemplo, modulo 3, sao sistemas
completos de residuos:
{0;1;2}, ou {3;4;5}, ou {-27;7;-1}, etc.
JP




-Mensagem original-
De: Marcos Eike Tinen dos Santos [EMAIL PROTECTED]
Para: [EMAIL PROTECTED] [EMAIL PROTECTED]
Data: Quarta-feira, 8 de Novembro de 2000 02:19
Assunto: Re: Dúvida sobre Resíduos


Ninguém Está querendo ajudar?
Enviei minha mensagem a algum tempo, e apena o Sr. JP me respondeu, assim
mesmo a segunda questão.

Ats,
Marcos Eike







Re: ajuda

2000-11-06 Por tôpico José Paulo Carneiro



A 1a equacao estah certa, mas nas contas, achei 
5.
JP

-Mensagem original-De: 
josimat [EMAIL PROTECTED]Para: 
[EMAIL PROTECTED] [EMAIL PROTECTED]Data: 
Segunda-feira, 6 de Novembro de 2000 00:42Assunto: Re: 
ajuda
Oi Filho!
500*1,65^n=125*2,178^n, com n 
medido em anos.
20=1,32^n
n=(log20)/(log1,32)=~10,8.
[]'s JOSIMAR

  -Mensagem original-De: 
  Filho [EMAIL PROTECTED]Para: 
  discussão de problemas [EMAIL PROTECTED]Data: 
  Domingo, 5 de Novembro de 2000 23:03Assunto: 
  ajuda
  Suponha que um assalariado ganha 500 unidades 
  monetárias mensalmente, com reajuste de 65% anual, e pague uma prestação de 
  125 unidades monetárias mensais, com reajuste anual de 117,8%. supondo fixos 
  esses reajustes, em quanto tempo, aproximadamente, o seu vencimento terá um 
  valor exatamente igual ao da prestação? Dados: ( log 4 = 0,60 e log 1,32 = 
  0,12 )


Re: O espaço outra vez

2000-11-03 Por tôpico José Paulo Carneiro


-Mensagem original-
De: David Pereira [EMAIL PROTECTED]
Para: [EMAIL PROTECTED] [EMAIL PROTECTED]
Data: Sexta-feira, 3 de Novembro de 2000 20:38
Assunto: Re: O espaço outra vez



a fórmula da distancia de um ponto a um plano em R³
( |aXo + bYo + cZo + d| ) / sqrt (a^2 + b^2)

=Faltou a parcela c^2 dentro da raiz quadrada.

é a distância entre um plano ax + by +cz + d = 0 e um ponto P(Xo,Yo,Zo).
x^2
é x ao quadrado e sqrt(x) é a raiz quadrada de x. Note a semelhança com a
equação da distância entre ponto e reta no R².

de um ponto a uma reta (também em R³)

Sejam Po (Xo; Yo; Zo) um ponto qualquer, Pr (Xr; Yr; Zr) um ponto da reta r
e seja V um vetor paralelo a r, a distância entre Po e r é:

|| (PrPo X V) || / ||V||²

Onde PrPo é o vetor entre Pr e Po, e ||x|| é a norma de um vetor x. V X U é
o produto vetorial entre V e U.

como se transformam coordenadas polares em R³ para (x;y;z).
Um ponto em Coordenadas Cilíndricas (Polares no R³) = (r,ø,z).
Transformando
para Coordenadas Cartesianas, temos um ponto (x,y,z) tal que:
x = r cos ø
y = r sen ø
z = z

[]s
David






Re: Desentendimentos e ângulo sólido

2000-11-03 Por tôpico José Paulo Carneiro




Eh facil ver que o produto vetorial nao eh 
associativo. Imagine vetores a, b, c tais que
(imaginando-os todos com origem na origem de 
R^3):
b eh paralelo a OX, a 
eh paralelo a bissetriz entre OX e OY no plano XOY , e c eh paralelo a 
OZ.
Entao axb eh paralelo a c, de modo que (axb)xc eh o vetor 
nulo, enquanto
bxc eh paralelo a OY, de modo que ax(bxc) eh nao nulo e 
paralelo a OZ.
JP

-Mensagem original-De: 
Jorge Peixoto Morais [EMAIL PROTECTED]Para: 
[EMAIL PROTECTED] [EMAIL PROTECTED]Data: 
Sexta-feira, 3 de Novembro de 2000 21:21Assunto: Desentendimentos 
e ngulo slido
*Nicolau (O Grande) disse que R no 
 associativo em relao ao produto vetorial, mas o Ralph 
parece discordar! Que negcio  esse?
*Em uma apostila estava escrito que a 
congruncia s funciona com nmeros inteiros, mas em outra 
eu encontrei 3/2 = -2 (mod 7), porque se ambos os lados forem multiplicados por 
2, fica 3 = -4 (mod 7). Que negcio  esse?

Agora o ngulo slido. ngulo 
slido  definido como o quociente entre a rea 
(determinada pelos planos que definem o angulo e a superficie da esfera) de uma 
esfera com centro em seu vrtice e a rea de superficie total da 
esfera?Eu pergunto isso porque parece que o angulo linear foi definido assim, 
mas substituindo esfera por circunferencia. 



Re: ajuda

2000-10-30 Por tôpico José Paulo Carneiro




O metodo eh geral, e baseia-se no fato de que se 
voce pensar nos graficos idade x tempo de quaisquer duas pessoas, voce obterah 
duas paralelas a bissetriz dos quadrantes impares (sem perda de generalidade, um 
deles pode ser a propria). Estas linhas chamam-se em Demografia linhas de 
vida. 
JP

-Mensagem original-De: 
josimat [EMAIL PROTECTED]Para: 
[EMAIL PROTECTED] [EMAIL PROTECTED]Data: 
Segunda-feira, 30 de Outubro de 2000 14:18Assunto: Re: 
ajuda
Ol Professor JP!
Suspeito de que o problema proposto pelo Carlos 
seja bem diferente do clssico problema apresentado na RPM 16, mas vou 
tentar aplicar o mtodo sugerido l.
[]'s JOSIMAR

-Mensagem original-De: 
Jos Paulo Carneiro [EMAIL PROTECTED]Para: 
[EMAIL PROTECTED] [EMAIL PROTECTED]Data: 
Segunda-feira, 30 de Outubro de 2000 13:05Assunto: Re: 
ajuda
A respeito de problemas de idades, sugiro o 
artigo da Revistado Professor de Matematica, numero 16:
Uma solucao 
geometrica para o problema das idades.
JP

-Mensagem original-De: 
josimat [EMAIL PROTECTED]Para: 
[EMAIL PROTECTED] [EMAIL PROTECTED]Data: 
Domingo, 29 de Outubro de 2000 21:11Assunto: Re: 
ajuda
Oi Carlos!
Pensei que conhecesse 
todos os problemas de idade, mas este  de deixar tonto.
A resposta do 
nmero 2 :
ANTONIO 27,5 anos e PEDRO 16,5 anos.
Resoluo:
Tente equacionar de baixo pra cima, voce vai desenrolar 
tudo:
A - idade atual de ANTNIO
P - idade atual de PEDRO
Considere que h x anos a idade de ANTONIO era o 
triplo da idade de PEDRO.
Entao:

A-x=3(P-x)
A=2[P-(3x-A)/2]
A+P=44
Donde vem x=11, A=27,5 e P=16,5, se no errei em 
nada. Por favor, confira.
Nao se acanhe em retornar caso nao tenha entendido, por 
favor.

[]'s JOSIMAR

-Mensagem 
original-De: Carlos Roberto de Moraes [EMAIL PROTECTED]Para: 
[EMAIL PROTECTED] 
[EMAIL PROTECTED]Data: 
Domingo, 29 de Outubro de 2000 14:44Assunto: 
ajuda
Gostaria de pedir ajuda para resolver os 
dois problemas abaixo:


1(FAAP) Um pas est 
lanando sua nova moeda, o royal, feita de uma liga 
de zinco e cobre. A Casa da Moeda dispe de duas ligas: numa , os 
metais esto na razo 2/3; na outra, esto na 
razo 3/7. Para cunhar as moedas, quer-se produzir 8 toneladas de 
uma nova liga em que os metais estejam na razo 5/11. Para tanto, 
devem ser usadas da primeira e da segunda ligarespectivamente, as 
quantidades( em toneladas):
a) 2 e 6 b) 3,5 
e 4,5 c) 4 e 4 d) 1,5 e 6,5 e) 1 e 
7



2) A soma das idades atuais de Pedro e 
Antonio  exatamente 44 anos. Antonio tem o dobro da idade que 
Pedro tinha quando Antonio tinha a metade da idade Que Pedro ter 
quando Pedro tiver 3 vezes a idade que Antonio tinha quando Antonio era 
tres vezes mais velho que Pedro. Qunatos anos ( e meses) tem cada 
um?


Se alguem puder me ajudar, desde 
j agradeo.


Re: Combinações afins e vetores transportadores no espaço

2000-10-30 Por tôpico José Paulo Carneiro




Claro que vale!
JP

-Mensagem original-De: 
Jorge Peixoto Morais [EMAIL PROTECTED]Para: 
[EMAIL PROTECTED] [EMAIL PROTECTED]Data: 
Segunda-feira, 30 de Outubro de 2000 20:32Assunto: 
Combinaes afins e vetores transportadores no 
espao
No espao tambm vale A-B= vetor AB 
(imagine uma setinha em cima de AB apontando para a direita)e Aa +Bb+ Cc+...+Zz= 
A + bAB + cAC+...+zAZ em que as letras minsculas so 
coeficientes, as maisuclas so pontos, duas letras 
maisculas juntas so um vetor e todos os coeficientes somam 
1?


Re: Vetores no espaço (talvez eu devesse comprar um bom livro; mas qual?)

2000-10-28 Por tôpico José Paulo Carneiro

OK, Nicolau.
Obrigado pela sua observacao. Nao foi um erro de tecla, foi um uma especie
de ato falho, por causa da apresentacao tradicional dos quaternions com i,
j, k (e mais o 1, eh claro, que fazem 4).
E por falar nisto, mais uma vez os complexos:
assim como o corpo dos complexos eh isomorfo ao das matrizes reais 2x2 da
forma (a;-b) (1a linha); (b;a) (2a linha),com a adicao e multiplicacao
usuais de matrizes,
os quaternions podem ser apresentados como as matrizes complexas 2x2 em que
a primeira linha eh (z ; -conj(w)) e a segunda linha (w ; conj(z)) (alguem
confira, pois estou citando de cabeca), e as operacoes usuais de matrizes.
Como cada complexo equivale a 2 reais, olha o R^4 ahi outa vez.
Interessante esta sua informacao sobre o R^8, que para mim eh novidade.
JP

-Mensagem original-
De: Nicolau C. Saldanha [EMAIL PROTECTED]
Para: [EMAIL PROTECTED] [EMAIL PROTECTED]
Data: Sexta-feira, 27 de Outubro de 2000 20:44
Assunto: Re: Vetores no espaço (talvez eu devesse comprar um bom livro; mas
qual?)




On Fri, 27 Oct 2000, José Paulo Carneiro wrote:

 Metendo minha colher no papo entre o Jorge e o Ralph:
 1) Voce pode definir quantas operacoes quiser com vetores, Jorge, mas eh
 claro que so levarao voce a serio se essas operacoes tiverem aplicacoes
 interessantes.
 2) A grande (imensa!) vantagem do produto de complexos eh que ela
 (juntamente com a adicao vetorial) torna o plano (algebricamente) um
corpo.
 E ja se sabe que nao eh possivel inventar multiplicacao semelhante em
nenhum
 R^n com n2 . O maximo que se consegue em R^3 eh um "quase corpo" (um
anel
 de divisao) em que a multiplicacao nap eh comutativa.

O JP provavelmente se distraiu ou errou de tecla:
quem tem estrutura de quase corpo é R^4, os quatérnios.
Os quatérnios são expressões da forma a + bi + cj + dk
onde a, b, c, d são reais, definimos a soma coordenada a coordenada
(i.e., (a + bi + cj + dk) + (e + fi + gj + hk) =
(a+e) + (b+f)i + (c+g)j + (d+h)k)
e a multiplicação por i^2 = j^2 = k^2 = -1,
ij = -ji = k, jk = -kj = i, ki = -ik = j
(assim, (a + bi + cj + dk) * (e + fi + gj + hk) =
(ae - bf - cg - dh) + (af + be + ch - dg)i +
(ag - bh + ce + df)j + (ah + bg - cf + de)k).

R^3 pode ser interpretado como o conjunto dos quatérnios de parte real
nula.
Neste caso o produto escalar é menos a parte real do produto
e o produto vetorial é a parte imaginária do produto.

Existe um produto não associativo importante em R^8;
com este produto os elementos de R^8 são chamados de octônios.
Estes (1,2,4,8) são os únicos valores de n para os quais R^n admite
um produto com certas propriedades legais
(*acho* que são distributividade em relação à soma dos dois lados,
conter uma cópia de R com as operações usuais e todo elemento não nulo
ter inverso multiplicativo).
O que eu sei com certeza é que R e C são os únicos R^n que são corpos
e que os quatérnios são o único R^n que é um "quase corpo".

[]s, N.






Re: Vetores no espaço (talvez eu devesse comprar um bom livro; mas qual?)

2000-10-27 Por tôpico José Paulo Carneiro

Metendo minha colher no papo entre o Jorge e o Ralph:
1) Voce pode definir quantas operacoes quiser com vetores, Jorge, mas eh
claro que so levarao voce a serio se essas operacoes tiverem aplicacoes
interessantes.
2) A grande (imensa!) vantagem do produto de complexos eh que ela
(juntamente com a adicao vetorial) torna o plano (algebricamente) um corpo.
E ja se sabe que nao eh possivel inventar multiplicacao semelhante em nenhum
R^n com n2 . O maximo que se consegue em R^3 eh um "quase corpo" (um anel
de divisao) em que a multiplicacao nap eh comutativa.
3) Mesmo assim, os complexos nao se dao por vencidos. O produto escalar de u
por v eh a parte real do produto (complexo) : conj(u). v, enquanto o produto
vetorial eh a parte imaginaria do mesmo produto conj(u).v, multiplicado pelo
unitario k.
JP


-Mensagem original-
De: Ralph Costa Teixeira [EMAIL PROTECTED]
Para: [EMAIL PROTECTED] [EMAIL PROTECTED]
Data: Quinta-feira, 26 de Outubro de 2000 23:54
Assunto: Re: Vetores no espaço (talvez eu devesse comprar um bom livro; mas
qual?)




 Jorge Peixoto Morais wrote:

 Antes de tudo: valeu, Ralph, pela atencao aa minha pergunta; seu
 e-mail foi extremamente instrutivo. Agora o principal: seu último
 e-mail me deixou com umas duvidas (se achar inconveniente me
 responder, me indique um bom livro):
 a) pelas regras que voce definiu, parece que mesmo atuando soh nos
 vetores em que z=0 (ou seja, no plano xy) as regras sao totalmente
 diferentes das que regem o plano dos complexos! Por que?

 Bom, sim, esse produto cartesiano a esse produto escalar realmente não
batem com o produto de números complexos quando z=0... Por quê? Bom,
para dizer a verdade, não esperaria que fossem o mesmo, de fato...

 a2) Vendo que essas regras sao diferentes das que regem o plano de
 Gauss, me pergunto: de onde, entao, elas vem?
 b)"ixj=-j. Mas isso nao eh perpendicular ao plano determinado por i e
 j!

 Oops... Se eu digitei isto, eu errei. Era pra ser ixj=k e ixk=-j.

 De onde elas vem... Bom, eu não sei historicamente onde que elas
surgiram... Mas eu costumo pensar assim: quando eu tento arrumar a
fórmula para o ângulo entre dois vetores, a conta u1v1+u2v2+u3v3
aparace; quando eu tento achar a projecao de u na direcao de v, a conta
acima tambem aparece; depois de achar um monte de lugares onde ela
aparece, eu resolvi dar um nome para ela para facilitar a minha vida: o
PRODUTO ESCALAR. Imagino algo semelhante para o produto cartesiano...
mas o fato é que a necessidade do conceito só parece intuitiva para
alguém *DEPOIS* que o conceito é bastante usado... Se alguém souber
melhor, favor me ajudar aqui. :)

 Na minha cabeça, produto escalar é uma ferramenta para achar ângulos
entre vetores, e o produto cartesiano para achar a área de seu
paralelogramo. *Começa* assim, e depois você vai achando um monte de
outras utilidades...

 Abraço,
 Ralph

 Mais uma vez, obrigado pelo trabalho de me escrever e-mails tao longos
 (mas com uma enorme densidade de informacao)





Re: Gauss-Seidel - 2

2000-10-23 Por tôpico José Paulo Carneiro





-Mensagem original-De: 
Humberto Ferreira Vinhais [EMAIL PROTECTED]Para: 
Olimpada de Matemtica [EMAIL PROTECTED]Data: 
Segunda-feira, 23 de Outubro de 2000 20:46Assunto: Gauss-Seidel - 
2

 Bem, sei que j enviei esse 
problema, mas  de muita importncia para mim resolv-lo e 
aps tentar muito, abrindo as situaes para n=3, 
no cheguei a nenhuma concluso lgica. Portanto, quanquer 
ajudazinha que qualuqer um de vcs possam me enviar, seria til, por 
exemplo: algumas implicaes de a matriz ser simtrica e de 
ser definida positiva, 

= Os fatos mais conhecidos 
aqui sao: uma matriz real simetrica tem todos os seus autovalores reais; 
autovetores correspondentes a autovalores diferentes sao mutuamente ortogonais; 
vale um tipo de teorema espectral: a matriz pode ser escrita como 
soma de parcelas do tipo a^k E_k, onde os a sao os autovalores da matriz, e os E 
sao matrizes que representam projecoes ortogonais, isto eh, sao 
idempotentes (E^2=E) e mutuamente ortogonais (E_i*E_k=0, quando i diferente de 
k). Estes fatos estao em qualquer compendio de matrizes. mas o que tem isto a 
vercom o Gauss-Seidel? Eh que em qualquer desses metodos iterativos para 
resolver sistemas de modo aproximado, o erro e_k no k-esimo passo se 
expressa em funcao do k-1 esimo passo como um produto matricial e_k=B*e_(k-1), o 
que implica e_n=B^n e_0, e ahi usa-se a forma espectral citada do B_n, a qual 
envolve as potencias dos autovalores. Em geral, uma condicao boa para 
convergencia eh algo do tipo: o autovalor de maior modulo tem modulo menor que 
1, pois entao se recai na convergencia de uma certa PG de razao entre 0 e 1. A 
condicao que voce cita, eu nem sabia que era suficiente (e continuarei duvidando 
ateh ver a demonstracao). O Gilbert Strang (Linear Algebra and its Applications) 
fala de uma condicao semelhante (sem demonstrar) mas exige tambem a positividade 
dos elementos da diagonal principal. Veja tambem livros de Calculo numerico, do 
tipo Algebra Linear Computacional (nao tenho aqui d cabeca um).
JP

ou algum detalhe que implique em algum teorema sobre matrizes 
ou algebra linear a partir da definio, enfim, qualquer coisa 
serve, mesmo que sej s a indicao de um livro que 
tenha algo relacionado, pois sei que existe um Teorema que  exatamente 
esse problema (ou seja, tirando o mostre que e afirmando ser 
verdade). Por favor.

Problema

Considere o sistema linear Ax=b , A pertencente a Mat_n ( IR 
), x,b pertence a IRn . Suponha que a matriz A  simtrica (isto 
, aij = aji ,quaisquer i, j ) e definida positiva (isto 
, Ax,x 0 para qualquer x de IRn - 
{0} ). Mostre que o Mtodo de Gauss-Seidel aplicado a este sistema, 
converge para a raiz.



Re: senos e cossenos

2000-10-19 Por tôpico José Paulo Carneiro

Nao eh possivel para todos os senos e cosenos.
Para 36gr, observe que num triangulo isosceles com 36gr no vertice
diferente, os outros angulos sao 72gr. Tracando uma bissetriz de um desses
angulos de 72, voce obtem outro trianglinho semelhante ao grandao. Com esta
observacao, voce mata o problema.
JP

-Mensagem original-
De: Marcelo Souza [EMAIL PROTECTED]
Para: [EMAIL PROTECTED] [EMAIL PROTECTED]
Data: Quinta-feira, 19 de Outubro de 2000 14:54
Assunto: senos e cossenos


Oi pessoal,
Como faço para obter senos e cossenos em radicais reais? Por exemple sen36?
obrigado
abraços
marcelo
_
Get Your Private, Free E-mail from MSN Hotmail at http://www.hotmail.com.

Share information about yourself, create your own public profile at
http://profiles.msn.com.






Re: Teorema de Napoleão

2000-10-18 Por tôpico José Paulo Carneiro




Duasobservacoes:
1) O Teorema de Napoleao que conheco eh com 
baricentros.

2) Os complexos foram 
descobertos no inicio do sec.XVI em um contexto algebrico.
Durante 300 anos, foram tratados como alucinacoes, 
imaginarios, etc.
Por volta de 1800, Wessel, Argand e, principalmente Gauss, 
comecaram a mostrar
que os complexos sao tao reais quanto os reais. Na realidade, 
fornecem ao plano uma estrutura de corpo (como a dos reais na reta), o que 
facilita tremendamente o tratamento algebrico (sem precisar apelar para 
coordenadas) da geometria plana, principalmente quando estao envolvidos angulos 
e rotacoes (o mesmo vale para a Fisica, quando sao envolvidos fenomenos 
periodicos).
Ainda tenho esperanca de que um dia (talvez daqui a mais 200 
anos, quem sabe?) se perca esta
implicancia com os numeros complexos, abolindo toda esta 
nomenclatura de complexos, imaginarios, etc., e 
simplesmente se defina no plano um produto maravilhoso que, junto com a adicao 
vetorial tradicional, faca do plano um corpo, onde se trabalhe com a mesma 
tranquilidade com que se trabalha com os reais na reta.
JP



-Mensagem original-De: 
Rodrigo Villard Milet [EMAIL PROTECTED]Para: 
[EMAIL PROTECTED] [EMAIL PROTECTED]Data: 
Tera-feira, 17 de Outubro de 2000 23:52Assunto: Re: 
Teorema de Napoleo
D uma olhada nessa 
resoluo por geometria sem complexos... t sem figura, mas 
d... eu j mandei uma vez essa pra lista !

Generalizao do Teorema de Napoleo : 
Dado um tringulo qualquer ABC, constroem-se os tringulos 
ABP, ACQ e BCR, todos semelhantes e exteriores a ABC. O tringulo formado 
pelos circunscentros dos tringulos exteriores  semelhante a ABC 
!
Lema : Os crculos circunscritos a ABP, ACQ e BCR passam por um 
mesmo ponto.
Prova do lema : Traando os crculos circunscritos a ABP e 
ACQ, vemos que so secantes em A e em outro ponto N. Basta, ento, 
provar que #BCRN  inscritvel, pois B, C e R determinam um 
crculo. Seja ANxBC=M , ang(MNC)=ang(AQC) {pois #AQCN  
inscrito}... analogamente, ang(BNM)=ang(APB). Pela contruo da 
figura, de um modo simtrico, para no tirar a generalidade do 
problema, ang(APB)=ang(ABC) e ang(AQC)=ang(BAC) assim, ang(BNC) = ang(BNM) + 
ang(CNM) = ang(APB) + ang(AQC) = ang(BNC) = ang(ABC) + ang(BAC) =180  
ang(BCA) = ang(BNC) = 180o  ang(BRS) = #BNCR  
inscrito. (CQD)
..
Lema : A corda comum a dois crculos  perpendicular a reta 
que une os centros.
Prova do lema : Basta ver que os centros das circunferncias 
eqidistam dos pontos de concorrncia das circunferncias 
ento eles determinam a mediatriz da corda comum. (CQD)
..
Sejam, ento, O1, O2 e O3 os centros 
dos crculos em ABP, ACQ e BCR, respectivamente. E sejam tambm, T 
= O1O2xAN , U = O2O3xCN e V = 
O1O3x BN . Assim, #O1TNV  
inscritvel = ang(BNM) = ang(O2O1O3) 
= ang(ABC)... Analogamente, analisando #O2TNU e #O3UNV, 
vemos que ang(O1O2O3)=ang(BAC) e 
ang(O1O3O2)=ang(ACB) = 
O1O2O3  semelhante a ABC. (CQD)
Villard ! 


-Mensagem original-De: 
Hugo Iver Vasconcelos Goncalves [EMAIL PROTECTED]Para: 
[EMAIL PROTECTED] [EMAIL PROTECTED]Data: 
Tera-feira, 17 de Outubro de 2000 20:44Assunto: 
Teorema de Napoleo
Ser que algum poderia mandar uma 
demonstrao do teorema de Napoleo ou dizer uma 
pgina onde posso 
encontr-la???


Re: Alguém pode dar uma explicação em algumas dúvidas?

2000-10-18 Por tôpico José Paulo Carneiro

O teorema de Euler supoe que a seja relativamente primo com n.
Uma maneira que acho interessante (nao sei se eh esta que estah na Eureka)
eh a seguinte: Considere U_n = {b_1, ..., b_k} = conjunto dos naturais entre
1 e n que sao primos com n. Por definicao de phi: k=phi(n). Considere os
numeros
a*b_1, ..., a*b_k e ache os seus restos de divisao por n (chamemos esses de
r_1, ..., r_k). Afirmo que todos os r_i sao diferentes, pois se, por
exemplo, a*b_1 e a*b_2 deixassem o mesmo resto por n, eles seriam congruos
mod n, e entao n seria um divisor de a*(b_1-b_2). Mas como a eh primo com n,
isto obrigaria n a dividir b_1-b_2, ou sejam b_1 e b_2 seriam congruos mod
n. Masisto eh imposssivel, pois os restos desses por n sao eles mesmos, e
eles sao diferentes.
Nao eh dificil ver que os r_i tambem sao primos com n (um fator comum a r_i
e n seria divisor de b_i).
Mas entao os r_i sao exatamente os b_i (possivelmente em outra ordem).
Agora, o produto dos a*b_i (que eh a^k vezes o produto dos b_i)  eh congruo
do produto dos r_i. Simplificando o produto dos r_i (que eh o mesmo dos
b_i), chega-se a: a^k congruo de 1 c.q.d.
Veja se os detalhes estao OK.
JP



-Mensagem original-
De: Marcos Eike Tinen dos Santos [EMAIL PROTECTED]
Para: [EMAIL PROTECTED] [EMAIL PROTECTED]
Data: Quarta-feira, 18 de Outubro de 2000 23:06
Assunto: Alguém pode dar uma explicação em algumas dúvidas?


Dúvidas sobre física e matemática.

1) Como se prova matematicamente a existência dos orbitais, aprendemos no
colégio de uma forma direta e "confusa", pois não sabemos de onde aparece
os
formatos etc.

2)Como posso provar fi de Euler: a^[fi(n)] == 1 (mod n)? Eu vi a prova no
Eureka 2, porém não a entendi por completo, pois há algumas transformações
que me pareceram "mágicas". :)

3) Problema:

Prove que dado n pertence N existe um conjunto de n  elementos   A está
contido em N tal que para todo B está contido em A, B diferente de vazio, a
somatória de x talque x pertence a B é uma potência não trivial (isto é, um
número da forma m^k, onde m, k  são inteiros maiores ou iguais a 2), ou
seja, A = {x_1, x_2,. x_n} tal que x_1, x_2,.x_n, x_1 + x_2, x_1 +x_3,., ,
.,x_1 + x_2 +.x_n  são todos potências não triviais.








Re: Livros de Geometria do Wagner

2000-10-17 Por tôpico José Paulo Carneiro

Oi Paulo.
Com a palavra, o autor, o Luis.
Abracos.
JP

-Mensagem original-
De: Paulo Santa Rita [EMAIL PROTECTED]
Para: [EMAIL PROTECTED] [EMAIL PROTECTED]
Data: Terça-feira, 17 de Outubro de 2000 17:21
Assunto: Re: Livros de Geometria do Wagner


Ola Prof Jose Paulo,

1) Considerando que nunca pude fazer um curso sistematico de
construcoes geometricas e que tal lacuna na formacao de quem
pretende ser um Matematico e insustentavel.

2) Considerando que o autor - Luis Lopes -  e um membro de
nossa lista que muito tem contribuido para enriquece-la

3) CONDIERANDO SOBRETUDO que o livro recebeu o conceito
EXCELENTE do Sr, que sem duvida nenhuma, sabe muitissimo bem
sobre o que fala.

4) Considerando que o idioma nao e problema

concluo que  estou, a principio, muito interessado neste
livro ! Mas, o eterno problema : e um livro caro ? O Luis
Lopes nao poderia facilitar o "acesso financeiro" a esta
obra ?

Um Abraco
Paulo Santa Rita
3,1536,17102000

On Mon, 16 Oct 2000 22:35:53 -0200
"=?iso-8859-1?Q?Jos=E9_Paulo_Carneiro?="
[EMAIL PROTECTED] wrote:
Eu conheco o liro do Luis (em frances) e posso garantir
que eh
excelente.
Jose Paulo

-Mensagem original-
De: Luis Lopes [EMAIL PROTECTED]
Para: [EMAIL PROTECTED] [EMAIL PROTECTED]
Data: Segunda-feira, 16 de Outubro de 2000 20:50
Assunto: Re: Livros de Geometria do Wagner


Sauda,c~oes,

Considerando que o assunto geometria e tri^angulos
interessa a muita
gente na lista, gostaria de dizer que escrevi um livro em
franc^es sobre
constru,c~oes com r'egua e compasso de tri^angulos
envolvendo todos
os casos poss'iveis com ^angulos, lados, cevianas
(alturas, bissetrizes e
medianas) e raios (inscrito, circunscrito e exinscritos).
S~ao 371 (se n~ao
estou enganado) problemas, todos resolvidos e com
figuras.
Os dados do primeiro s~ao os tr^es ^angulos
(\alpha,\beta,\gamma) e os
do 'ultimo, os tr^es raios exinscritos (r_a,r_b,r_c).

A m'edio prazo estou pensando em coloc'a-lo em portugu^es
tamb'em, assim
como fazer o Volume 2 envolvendo combina,c~oes daqueles
elementos. Em
particular, os problemas (\alpha,a+b,a+c) e
(\alpha,a+c,b+c) s~ao muito
interessantes e deixo como exerc'icios para os membros.

Pe,co aos interessados em ter mais detalhes que escrevam
diretamente para
mim.
Obrigado.

[ ]'s
Lu'is

-Mensagem Original-
De: Eduardo Wagner [EMAIL PROTECTED]
Para: [EMAIL PROTECTED]
Enviada em: Terça-feira, 17 de Outubro de 2000 02:26
Assunto: Re: Livros de Geometria do Wagner


Caros amigos:
Fico feliz em saber que algo que publiquei ainda jovem,
influenciou pessoas
e ainda hoje se procura. Na verdade, nada se publicou de
forma séria em
geometria desde então. O livro Geometria II será
inteiramente refeito em
2001, com erros corrigidos e ampliado. O espírito não
será modificado, ou
seja, o livro não será dedicado ao currículo normal das
escolas mas sim
àqueles que desejam conhecer mais sobre a geometria. O
trabalho será grande
e vai demandar algum tempo.
Abraços,
Wagner.








Don't E-Mail, ZipMail! http://www.zipmail.com/





Re: Teorema de Napoleão

2000-10-17 Por tôpico José Paulo Carneiro




Eureka 6, artigo do Edmilson Motta.
JP

-Mensagem original-De: 
Hugo Iver Vasconcelos Goncalves [EMAIL PROTECTED]Para: [EMAIL PROTECTED] [EMAIL PROTECTED]Data: 
Tera-feira, 17 de Outubro de 2000 21:30Assunto: Teorema 
de Napoleo
Ser que algum poderia mandar uma 
demonstrao do teorema de Napoleo ou dizer uma 
pgina onde posso encontr-la???


Re: Livros de Geometria do Wagner

2000-10-16 Por tôpico José Paulo Carneiro

Eu conheco o liro do Luis (em frances) e posso garantir que eh
excelente.
Jose Paulo

-Mensagem original-
De: Luis Lopes [EMAIL PROTECTED]
Para: [EMAIL PROTECTED] [EMAIL PROTECTED]
Data: Segunda-feira, 16 de Outubro de 2000 20:50
Assunto: Re: Livros de Geometria do Wagner


Sauda,c~oes,

Considerando que o assunto geometria e tri^angulos interessa a muita
gente na lista, gostaria de dizer que escrevi um livro em franc^es sobre
constru,c~oes com r'egua e compasso de tri^angulos envolvendo todos
os casos poss'iveis com ^angulos, lados, cevianas (alturas, bissetrizes e
medianas) e raios (inscrito, circunscrito e exinscritos). S~ao 371 (se n~ao
estou enganado) problemas, todos resolvidos e com figuras.
Os dados do primeiro s~ao os tr^es ^angulos (\alpha,\beta,\gamma) e os
do 'ultimo, os tr^es raios exinscritos (r_a,r_b,r_c).

A m'edio prazo estou pensando em coloc'a-lo em portugu^es tamb'em, assim
como fazer o Volume 2 envolvendo combina,c~oes daqueles elementos. Em
particular, os problemas (\alpha,a+b,a+c) e (\alpha,a+c,b+c) s~ao muito
interessantes e deixo como exerc'icios para os membros.

Pe,co aos interessados em ter mais detalhes que escrevam diretamente para
mim.
Obrigado.

[ ]'s
Lu'is

-Mensagem Original-
De: Eduardo Wagner [EMAIL PROTECTED]
Para: [EMAIL PROTECTED]
Enviada em: Terça-feira, 17 de Outubro de 2000 02:26
Assunto: Re: Livros de Geometria do Wagner


Caros amigos:
Fico feliz em saber que algo que publiquei ainda jovem, influenciou pessoas
e ainda hoje se procura. Na verdade, nada se publicou de forma séria em
geometria desde então. O livro Geometria II será inteiramente refeito em
2001, com erros corrigidos e ampliado. O espírito não será modificado, ou
seja, o livro não será dedicado ao currículo normal das escolas mas sim
àqueles que desejam conhecer mais sobre a geometria. O trabalho será grande
e vai demandar algum tempo.
Abraços,
Wagner.








Re: Lei do Corte

2000-10-04 Por tôpico José Paulo Carneiro

Sai dapropria proposicao direta:
m+p  n+p =  (m+p)+(-p)  (n+p)+(-p)
o que eh o mesmo que m  n


-Mensagem original-
De: Marcelo Souza [EMAIL PROTECTED]
Para: [EMAIL PROTECTED] [EMAIL PROTECTED]
Data: Quarta-feira, 4 de Outubro de 2000 14:15
Assunto: Re: Lei do Corte


oi, essa lei do corte saiu da proposiç]ao que mn = m+pn+p, pede pra
provar que a reciproca desta proposiç]ao usando absurdo, monotonicidade e
tricotomia.
espero que seja isso...
abraços
marcelo

From: "José Paulo Carneiro" [EMAIL PROTECTED]
Reply-To: [EMAIL PROTECTED]
To: [EMAIL PROTECTED]
Subject: Re: Lei do Corte
Date: Tue, 3 Oct 2000 21:08:11 -0300

Eh preciso primeiro esclarecer a que lei do corte voce se refere:
a de uma parcela na adicao de numeros (que numeros?),
a de um fator na multiplicacao (de numeros? quais?),
e ainda quais sao os pressupostos dos quais se parte.

-Mensagem original-
De: Marcelo Souza [EMAIL PROTECTED]
Para: [EMAIL PROTECTED] [EMAIL PROTECTED]
Data: Terça-feira, 3 de Outubro de 2000 13:42
Assunto: Lei do Corte


 Oi pessoal
 Alguém poderia me ajudar? Como posso provar a Lei do Corte por absurdo,
 usando a tricotomia e a monotonicidade?
 Abraços
 Obrigado
 Marcelo

_
 Get Your Private, Free E-mail from MSN Hotmail at
http://www.hotmail.com.
 
 Share information about yourself, create your own public profile at
 http://profiles.msn.com.
 
 


_
Get Your Private, Free E-mail from MSN Hotmail at http://www.hotmail.com.

Share information about yourself, create your own public profile at
http://profiles.msn.com.






Re: Lei do Corte

2000-10-03 Por tôpico José Paulo Carneiro

Eh preciso primeiro esclarecer a que lei do corte voce se refere:
a de uma parcela na adicao de numeros (que numeros?),
a de um fator na multiplicacao (de numeros? quais?),
e ainda quais sao os pressupostos dos quais se parte.

-Mensagem original-
De: Marcelo Souza [EMAIL PROTECTED]
Para: [EMAIL PROTECTED] [EMAIL PROTECTED]
Data: Terça-feira, 3 de Outubro de 2000 13:42
Assunto: Lei do Corte


Oi pessoal
Alguém poderia me ajudar? Como posso provar a Lei do Corte por absurdo,
usando a tricotomia e a monotonicidade?
Abraços
Obrigado
Marcelo
_
Get Your Private, Free E-mail from MSN Hotmail at http://www.hotmail.com.

Share information about yourself, create your own public profile at
http://profiles.msn.com.






Re: Ajuda sobre Geometria Analítica

2000-10-03 Por tôpico José Paulo Carneiro

So uma observacao: a expressao (latina) correta eh:
latus rectum.

-Mensagem original-
De: Eduardo Quintas da Silva [EMAIL PROTECTED]
Para: [EMAIL PROTECTED] [EMAIL PROTECTED]
Data: Terça-feira, 3 de Outubro de 2000 19:16
Assunto: Re: Ajuda sobre Geometria Analítica


Obrigado Paulo pela dica vou correr atrás desse livro mas tenho um pouco
de pressa
Será que que posso encontrar esse livro na biblioteca do IMPA 
Aguem poderia me ajudar...
Valeu

Paulo Santa Rita escreveu:

 Ola Eduardo,

 Quando eu procurei compreender melhor estes temas, alguns
 anos atras, tambem encontrei a mesma dificuldade que voce
 demonstra estar encontrando ... A maioria dos livros so
 tratam das EQUACOES CANONICAS DAS CONICAS EM SUAS EXPRESSOES
 CARTESIANAS, vale dizer :

 1) Nao abordam a equacao geral do 2 grau em duas variaveis.

 2) Nao tratam das expressoes em coordenadas polares, o que
 permite uma compreensao unitaria de todas as conicas em
 funcao do conceito de excentricidade

 Tudo isso e muito chato e faz com que problemas que seriam
 triviais se soubessemos estas coisas se tornem dificeis tao
 somente em funcao de nossa ignorancia por nao termos tido
 acesso a boas obras ...

 a Grande Inteligencia e boa e eu tive a sorte de encontrar
 em um sebo ( mercado de livros velhos e usados ) a obra :

 Geometria Analitica
 de Nikolai Efimov
 Editora Mir

 Neste livro - Maravilhoso ! - voce vai encontrar tudo aquilo
 que e interessante, que nao encontra nos livros tradicionais
 e que tornara, doravante, os problemas de analitica que
 envolvem conicas todos muito simples. Nele voce vera tambem
 uma razoavel introducao as quadricas. Todavia, como tudo que
 e bom, e um livro dificil de ser encontrado ...

 Voce tambem pode ver uma expressao em algebra linear e
 vetores para as conicas. Consulte :

 Algebra Linear,
 de Boldrini ( e outros )
 Nao sei a editora

 Este livro e facilmente encontrado ... A vantagem de usa-lo
 e que voce, simultaneamente, tem uma amena introducao a
 algebra linear, o que sera importante posteriormente, quando
 voce for estudar Analise no R^n e Calculo Diferencial
 Exterior ( A derivada e, em verdade, uma matriz e, nao, um
 numero, como parece a principio ).

 O Prof Efimov tem um outro livro maravilhoso :

 Geometria Superior
 de Nikolay Efimov
 Editora Mir

 Nao ha dinheiro que paga este livro !

 Finalmente, so a titulo de Experiencia, consiga o livro de
 Geometria Analitica do Prof Efimov. Estude-o com afinco e
 dedicacao. Volte a seguir e considere os problemas mais
 dificeis sobre Conicas que caem nos Concursos, Vestibulares
 e Olimpiadas : voce passara a olha-los com outros olhos ...

 Um Abraco
 Paulo Santa Rita
 3,1545,03102000

 On Mon, 02 Oct 2000 22:44:50 -0300
 Eduardo Quintas da Silva [EMAIL PROTECTED] wrote:
 Gostaria que alguém me explicasse o conceito de reta
 diretriz para
 elipse e para hipérbole... consultando diversos livros
 didáticos ninguém
 escreve nada sobre isso. Eu sei que para a elipse as
 equaçoes sao : x =
 +- (a^2)/c
 Outra coisa que nao consigo achar e sobre o conceito de
 lactus rectum
 (ou qq coisa parecida com isso)...
 Obrigado...
 Eduardo
 


 
 Don't E-Mail, ZipMail! http://www.zipmail.com/








Re: Geometria analítica

2000-10-02 Por tôpico José Paulo Carneiro

A parabola pode ser definida como o conjunto (ou lugar geometrico) dos
pontos do plano que equidistam de um ponto dado (chamado foco) e uma reta
dada (chamada diretriz).  A distancia entre o foco e a diretriz eh o
parametro (alguns chamam de parametro a metade desta distancia).

-Mensagem original-
De: Douglas C. Andrade [EMAIL PROTECTED]
Para: Lista de Matemática [EMAIL PROTECTED]
Data: Segunda-feira, 2 de Outubro de 2000 22:00
Assunto: Geometria analítica


Que é um foco e um parâmetro de uma parábola?






Re: Triângulos...

2000-10-01 Por tôpico José Paulo Carneiro




A area de um triangulo eh a metade de bc sen A. 
Eleve ao quadrado, 
substitua sen^2 por 1-cos^2, e substitua cos A pela expressao que 
aparece na lei dos cosenos. O resto eh 
algebrismo.
JP

-Mensagem original-De: 
Pedro [EMAIL PROTECTED]Para: 
[EMAIL PROTECTED] [EMAIL PROTECTED]Data: 
Domingo, 1 de Outubro de 2000 10:12Assunto: 
Tringulos...
Algum poderiademonstrar aquela frmula 
para calcular a rea de um tringulo de lados a,b e 
csqrt(p(p-a)(p-b)(p-c))?



Re: vale o raciocínio

2000-09-25 Por tôpico José Paulo Carneiro

Pessoal:
Quando voces encontrarem uma expressao do tipo "infinito + infinito =
infinito", ela deve ser interpretada como uma expressao simplificada e
mnemonica de um teorema sobre limites, tal como: "se uma sequencia x de
numeros reais tende a infinito e uma sequencia y de numeros reais tende a
infinito, entao a sequencia x+y (que se obtem somando termo a termo) tambem
tende a infinito".

Isto nao significa que exista um numero, representado por 8 deitado, que
somado com ele mesmo de ele mesmo (so o numero 0 tem esta propriedade).

Por outro lado, quando aparece uma expressao do tipo "infinito - infinito eh
um caso de indeterminacao", isto significa apenas que nao existe teorema a
respeito desta situacao, ja que se podem encontrar exemplos (simples) de  x
tendendo a infinito e y tendendo a infinito, com x-y tendendo a 0, ou a 1,
ou a qualquer real r, ou a infinito, ou a -infinito, ou ateh nao tendo
limite.

Para dizer a verdade, eu particularmente acho que estas duas expressoes sao
maus habitos (que eu nao uso), e que frequentemente conduzem a confusao.
Acho preferivel escrever explicitamente suas versoes como limites, que eh o
que realmente sao. Alias, por que tanta preguica de escrever? nao tira
pedaco.

Passando ao caso: a sequencia (1+c/n)^n tende a e^c. Isto ja mostra que o
limite depende do c fixado, e no entanto, sempre 1+c/n tende a 1, e n tende
a infinito.
JP




-Mensagem original-
De: Benjamin Hinrichs [EMAIL PROTECTED]
Para: [EMAIL PROTECTED] [EMAIL PROTECTED]
Data: Sábado, 23 de Setembro de 2000 16:28
Assunto: vale o "raciocínio"


Amigos listeiros...
alguns já se inscreveream na ciencialist?? Espero que sim, a conversa lá
está boa. Para os que perderam, é uma lista de ciências da página do
eGroups
(www.egroups.com).

Vendo que foi usada uma notação de limite na lista, resolvi fazer a minha
linha pouco lógica de raciocínio.
Ora:
lim (1 + 1/x)^x = lim (1 + 0)^(oo) = lim 1 ^ (oo)
x - oo   x - oo   x - oo

Portanto 1 elevado na infinito não pode ser definido. Está correto esse
raciocínio?
Ou seria 1 diferente de 1,0...001 (= lim(1/x) com x - oo)

Abraço,


Benjamin Hinrichs







Re: N'umeros de Hamilton

2000-09-18 Por tôpico José Paulo Carneiro

O matematico que estabeleceu em bases solidas a Teoria das Distribuicoes foi
Laurent Scwartz, frances (se nao me engano, ganhador da Medalha Fields)
JP

-Mensagem original-
De: Paulo Santa Rita [EMAIL PROTECTED]
Para: [EMAIL PROTECTED] [EMAIL PROTECTED]
Data: Segunda-feira, 18 de Setembro de 2000 11:36
Assunto: N'umeros de Hamilton


Ola Luiz,
Tudo Legal ?

E a primeira vez que vejo uma mensagem sua em minha caixa de
correio. Voce deve ser novo na lista ... Bem-Vindo ! Prazer
em conhece-lo !

Eu me referi explicitamente a relacao entre V ( velocidade )
e X ( posicao ) porque :

1) Esta lista e acomopanhada por muitos alunos de nivel
medio e, neste nivel, a introducao a Fisica Quantica ( Por
Exemplo : Atomistica, de Feltre-Setsuo ) que e dada se
refere somente a esta duas grandezas

2) A maior parte da Fisica que e estudada pelo imensa
maioria dos alunos de graduacao em Matematica e Engenharia (
outro tipo de leitor desta lista )e  uma visao meramente
introdutória de Relatividade e Quantica.

Foi pensando nestes dois aspectos que eu citei
preferencialmente a aplicacao do principio das incertezas ao
binomio Velocidade X Posicao.

Eu fiz os cursos basicos de Fisica para um estudante de
Engenharia e, por interesse e curiosidade propria,
acompanhei cursos das Escolas de Fisica e Matematica. Os
livros em que alicercei meus CONHECIMENTOS BASICOS basicos
devem ser do conhecimento do Sr :

1) Curso de Fisica de Berkeley ( varios volumes, alguns em
ingles )
2) Introducao da Relatividade, de Resnick ( um volume )
3) Fisica Quantica, de Eilemberg ( Um volume, grosso )

A exposicao que ouvi da Teoria de Dirac usava, realmente, o
esquema de setas e era explicitamente referenciada pelo
professor como Diagrama das Setas. Parece-meque com isso ele
queria contrapor a exposicao de Feynman em que SE CONSIDERAM
AS VARIAS TRAJETORIAS POSSIVEIS ( aqui deve estar a origem
da palavra Historia ) e, a seguir, somam-se tudo, dando o
caminho mais provavel.

Inclusive, se nao me falha a memoria, foi assim que Feynman
expos sua Teoria a uma plateia que tinha, entre outros
Fisicos, o proprio Dirac, no MIT.

Eu fiquei interessado neste ponto que vista ( sobre o qual
sei pouco e que pretendo aprofundar ) de Feynman porque
achava intuitivo que o movimento fisico e individual de um
corpo que faz parte de um sistema deve ser participativo da
evolucao do sistema, objetivando o seu caminho mais
provavel, como na historia individual de um ser vivo que faz
parte de um eco-sistema.

Imagine que voce quer descrever a evolucao de uma Borboleta
( quon ). O movimento dela tem uma INDETERMINACAO INTRINSECA
( incerteza ) que nao nos pertuba, pois para os nossos
objetivos basta sabermos o LOCAL MAIS PROVAVEL ONDE ELA ESTA
( orbital ). Se, porem, voce quer descrever o movimento
fisico e individual dela, o seu caminhar indeterminado e um
problema e, muito provavelmente, voce vai buscar VARIAVEIS
OCULTAS que tornem determinado o que, por natureza, e
indeterminado. Mesmo que sua descricao de evolucao nao
acerte o destino de uma borboleta individual, a da maioria
voce vai acertar, ALEM DE PRESERVAR A INDIVIDUALIDADE que
elas, como seres vivos, merecem ter.

Isso e meramente uma ideia, sem maiores pretensoes, mas foi
o que me estimulou a estudar a formulacao de Feynmam. Eu
acho isso tudo muito interessante, mas e claro que tudo pode
nao passar de uma ideia maluca que vez ou outra nos visita :
e dificil voce estudar passivamente sem refletir sobre o que
esta aprendendo !

Voce ja leu "O Fim das Certezas", de Ylia Prigogine ? E um
respeitavel Prof, detentor de um Premio Nobel. O que voce
acha das ideias dele ?

Sobre Fisicos que Contribuem para a Matematica, concordo em
genero, numero e grau contigo ... Dentre as centenas de
Fisicos que a Humanidade ja teve e tem, alguns poucos
contribuiram para o Progresso da Matematica. A Maioria usa
aquilo que os Matematicos ja criaram.

Os exemplos que o Sr cita sao respeitaveis. O Calculo que
Newton criou, em parte para abordar os problemas derivados
de sua teoria gravitacional ( Para efeitos gravitacionais,
uma distribuicao esfero-simetrica de massa se comporta como
se toda a sua massa estivesse concentrada no centro
geometrico da esfera ) e a funcao delta de Dirac ( impulso
);

D(x) = 0, se x e diferende de zero
D(x) = infinito, se x = o
integral de D(x) de -inf a +inf = 1

Muito bem explicada e tornada manipulavel pela Teoria das
Distribuicoes de um grande Matematico Frances ( E Laurent
alguma
coisa ).

Em verdade, Matematica e Fisica deveriam ser uma unica
ciencia, dado as profundas ligacoes que elas guardam. Eu
posso estar errado, mas acho que muitas belas e importantes
conquistas matematicas nao seriam possiveis sem os problemas
suscitados pelo mundo fisico ...

Bom, o Sr deve ser um Prof de Fisica ou Estudante Avancado e
eu agradeco e estou aberto a qualquer ensinamento que o Sr
desejar me transmitir.

Bom, aqui e uma lista de discussao de problemas de
matematica ( preferencialmente de nivel medio ) e o 

Re: N'umeros de Hamilton

2000-09-15 Por tôpico José Paulo Carneiro

Ola Paulo,
obrigado por suas palavras sempre gentis.
Os livros de que falei sao importados, mas nao sao tao caros assim.
Voce tem razao no que disse a respeito dos livros de divulgacao,
que podem ser as vezes traicoeiros, mas eles sao necessarios.
O "Seis pecas faceis" eh uma coletanea de palestras feita pelo
Feynman para alunos de graduacao em Fisica
(se nao me engano no California Institute of Technology).
Ha muita coisa sobre sua favorita Mecanica Quantica.

O espaco eh muito pequeno aqui para entrar em detalhes, mas vou
dizer algo que talvez o decepcione um pouco: pessoalmente, acho
que o significado do teorema de Goedel tem sido as vezes deturpado,
como se significasse uma especie de "tocou bagunca" na Matematica, quando se
trata de algo eminentemente tecnico.
O mesmo se tem passado com o Caos e outras coisas semelhantes,
que tem caido nas maos  de leigos e tem sido usado com finalidades
"descontrucionistas" discutiveis.
Sobre isto, recomendo a leitura de  "Imposturas Intelectuais", de
Alan Sokal e R.Bricmont, ambos fisicos.
Um abraco grande, Paulo.
JP


-Mensagem original-
De: Paulo Santa Rita [EMAIL PROTECTED]
Para: [EMAIL PROTECTED] [EMAIL PROTECTED]
Data: Sexta-feira, 15 de Setembro de 2000 18:51
Assunto: Re: N'umeros de Hamilton


Ola Prof Jose Paulo,
Como vai ?

Fiquei muito feliz em receber esta sua mensagem. E a
primeira desde muito tempo. Como estao todos ?

E verdade. O nome correto e Feynman. Eu sempre cometo esses
erros simplorios, pois escrevo com certa pressa, dado que a
maquina que uso nao esta sempre disponivel.

Eu nao conheco esses livros de divulgacao a que o Sr se
refere. Eu tenho muito poucos livros deste tipo, pois eles
costumam ser muito caros. Mas vou ver se encontro algum(ns)
por aqui ( acho dificil ! ). Nao sei se o Sr concorda
comigo, mas o mal dos livros de divulgacao e que eles nao
nos permitem uma compreensao detalhada, ficando em geral so
"no papo". Eu, por enquanto, acho melhor estudar o assunto
"a vera", com material didatico, artigos e muitos
exercicios. E fazendo muitos exercicios e refletindo sobre
eles, investigando os limites de sua aplicabilidade,
compondo pequenos artigos e refazendo experiencias ( quando
possivel ) e demonstracoes que se aprende. Aprender e uma
luta !

Nao sei se se pode falar em "simpatia por um tema", mas o
certo e que as ideias quanticas sempre me atrairam muito e
mesmo antes de conhecer todas as implicacoes das relacoes de
incertezas ( e acredito que ainda nao conheca todas ) eu ja
era um firme adepto da tese holista e indeterminista. Ela
nos da uma maior liberdade de pensamento. Francamente,
depois do Teorema de Godel, me parece que a segunda maior
conquista da humanidade no seculo XX foi a mecanica
quantica.

O Teorema de Godel na Matematica e as relacoes de incertezas
na Fisica acabaram por completo com as pretensoes dos
formalistas e reducionistas de forma geral, abrindo caminho
para possibilidades insuspeitas. A esperanca em um futuro
melhor para a humanidae e muito mais certa com eles que nos
modelos anteriores.

A principio o Teorema de Godel foi quase completamente
ignorado,mas vem ganhando atencao gradativamente e tenho
lido muitos cerebros ilustres o apreciarem por diversos
angulos e quase ja consigo demonstra-lo por completo. Eu
imagino que doravante vao comecar a surgir mais e mais
exemplos de teoremas bastantes verossimeis e, no entanto,
indemonstraveis dentro dos axiomas e objetos tradicionais
dos diversos ramos da Matematica : isto mostrara a falencia
do metodo axiomatico e obrigara os Matematicos buscarem
novas formas de tratarem os problemas. O CARA LA DE CIMA nao
brinca e nao inspirou Godel para ele produzir um resultado
tao importante sem maiores intencoes ...

O mesmo se diga das relacoes de incertezas !

Esse papo e longo, mas, se Deus Quiser, nos teremos
oportunidades de discutir isso pessoalmente.

Um abracao pro Sr, e para os Prof Wagner, Morgado, Nicolau e
Gugu

Paulo Santa Rita
6,1813,15092000











On Fri, 15 Sep 2000 15:52:53 -0300
"=?iso-8859-1?Q?Jos=E9_Paulo_Carneiro?="
[EMAIL PROTECTED] wrote:
Caro Paulo:
O nome correto eh Feynman, mais precisamente,
Richard Feynman, fisico americano, tambem premio Nobel em
Fisica,
que esteve no Brasil. Ele tem livros de divulgacao sobre
todos esses
assuntos, tais como: "Six easy pieces" (6 pecas faceis) e
"Six not so
easy pieces" (6 pecas nao tao faceis).
JP

-Mensagem original-
De: Paulo Santa Rita [EMAIL PROTECTED]
Para: [EMAIL PROTECTED] [EMAIL PROTECTED]
Data: Sexta-feira, 15 de Setembro de 2000 10:21
Assunto: Re: N'umeros de Hamilton


Oi Leonardo,
Tudo Legal ?

Com relacao ao nome, voce esta certo  : O  correto e
"Heisenberg". Quanto a designacao do Principio, nao. O
nome
deste principio flutua ao sabor dos autores e e mesmo uma
questao de somenos importancia o problema do "nome
correto".
O que e importante e uma compreensao precisa do conteudo
factual do Principio.

Em Fisica Elementar este Principio costuma ser
apresentado
como segue : Nao e 

Re: Propriedades tautocrona e Braquistocrona da cicloide

2000-09-13 Por tôpico José Paulo Carneiro




Veja o site:

http://mathworld.wolfram.com/TautochroneProblem.html

JP


-Mensagem original-De: 
Marcos Paulo [EMAIL PROTECTED]Para: 
[EMAIL PROTECTED] [EMAIL PROTECTED]Data: 
Tera-feira, 12 de Setembro de 2000 14:07Assunto: 
Propriedades tautocrona e Braquistocrona da cicloide
Desculpem reenviar esta msg mas acredito que qdo 
enviei pela primeira vez havia muitas questoes ssobre a olimpiada Brasileira ... 
reenvio na esperanca de que alguem posa me ajudar. obrigado.

Alguem conhece uma demonstracao de que a curva com 
a propriedade tautocrona (e braquistocrona)  um arco de cicloide? Eu vi 
a pouco tempo uma demonstracao que usava transformada de Laplace .. esta, no 
entanto, nao me interessa ... se alguem puder ajudar me mostrando outra 
demonstracao ou alguma bibliografia agradeco.

[]'s MP


Re: Problema da olimpiada da america central e do caribe

2000-09-10 Por tôpico José Paulo Carneiro

Quero convidar os amigos da lista para aproveitar este problema para
refletir sobre uma questao pela qual tenho me batido: o (para mim absurdo)
abandodno a que foi relegado o ensino dos vetores no ensino medio.
Repare que, se colocarmos a origem em um ponto O qualquer e identificarmos
cada ponto X com o vetor definido pelo segmento orientado OX (e de modo
geral AB=B-A), teremos, para os baricentros F,G,H,I:
F=(A+B+E)/3;  G=(B+C+E)/3; etc. Dahi:
vetor FG=G-F=(C-A)/3=vetor AC/3
vetor IH=H-I=(C-A)/3=vetor AC/3
Logo, FGHI eh um paralelogramo.

(A questao da area eh mais elaborada, mas tambem pode sair por vetores: a
area do paralelogramo eh o modulo do produto vetorial de FG e GH.)

Mas o que me parece interessante eh que o problema so eh "sinistro" porque
perdeu-se o habito de usar vetores.
JP

Para: Lista de Matemática [EMAIL PROTECTED]
Data: Domingo, 10 de Setembro de 2000 00:12
Assunto: Problema da olimpiada da america central e do caribe


Vi um problema muito "sinistro" la:

Seja ABCDE um pentagono convexo. Sejam P, Q, R e S os baricentros dos
triangulod ABE, BCE, CDE e DAE, respectivamente.

Mostrar que PQRS e um paralelogramo e que sua area e igual a 2/9 da area do
quadrilatero ABCD.

(para os que quiserem, abaixo vai o problema escrito no original)

Sea ABCDE un pentagono convexo (las diagonales quedan dentro del
pentágono).
Sean P, Q, R y S los baricentros de los triangulos ABE, BCE, CDE y DAE,
respectivamente.

Demostrar que PQRS es un paralelogramo y que su area es igual a 2/9 del
área
del cuadrilátero ABCD.

Nota: El baricentro o centroide es el punto donde concurren las medianas.








En: RPM polêmica

2000-09-09 Por tôpico José Paulo Carneiro


-Mensagem original-
De: José Paulo Carneiro [EMAIL PROTECTED]
Para: José Paulo Carneiro [EMAIL PROTECTED]
Data: Sábado, 9 de Setembro de 2000 11:37
Assunto: Re: RPM polêmica


Desculpe: houve um problema, e a mensagem saiu totalmente truncada.

-Mensagem original-
De: José Paulo Carneiro [EMAIL PROTECTED]
Para: [EMAIL PROTECTED] [EMAIL PROTECTED]
Data: Sábado, 9 de Setembro de 2000 10:25
Assunto: Re: RPM polêmica


Muito interessante esta sua duvida.
Eh isto mesmo. Existem construcoes que nao dependem da unidade escolhida,
e
outras que dependem.
O primeiro caso ocorre quando a expressao que traduz a incognita em termos
dos dados eh "homogenea", e a segunda ocorre no caso contrario.
Por exemplo, a construcao da media geometrica de 2 segmentos nao depende
de
unidade, pois dados a e b, a m.g. x=RQ(ab). Se a e b representam as
medidas
dos segmentos em questao na unidade u, uma mudanca para a unidade v tal
que
u=tv farah com que os segmentos se escrevam au= (at)v e bu= (bt)v, ou
sejam,
as novas medidas sao at e bt, de modo que a nova media geometrica eh:
RQ(atbt)=t RQ(ab)=tx. Ou seja, o segmento resultante tem medida tx na
unidade v. Portanto, ele eh (tx)v=xu, que eh o original.
O mesmo ocorre com a quarta proporcional, x=(ab)c.
Ja se x=ab ou x=a^2 ou x=RQ(a), as expressoes nao sao homogeneas, e
portanto
dependem da unidade, o que fica claro pela propria construcao.
JP





-Mensagem original-
De: Jorge Peixoto Morais [EMAIL PROTECTED]
Para: [EMAIL PROTECTED] [EMAIL PROTECTED]
Data: Sexta-feira, 8 de Setembro de 2000 19:57
Assunto: RPM polêmica


Este e-mail é meio longo... mas é interessante.
 De novo sobre a RPM...Eu ainda não sei aquelas coisas do
primeiro
e-mail... E, adicionalmente, na mesma RPM...
Vocês viram aquele método para se calcular o quadrado de um
segmento?
Ele diz que para calcularmos a raiz quadrada de um segmento x,
escolhemos um segmento unitário qualquer, e construímos um triângulo
retângulo em que a altura da hipotenusa é x e a projeção de um dos
catetos
sobre a hipotenusa é 1, ou seja, um segmento que se diz unitário. Então,
pelas relações métricas, a outra projeção deve ser X².Aí é que está o
"problema".
É claro que se trocarmos o "segmento de comparação" U por, por
exemplo, 2U, o segmento que media X "Us" vai medir (X/2)*("2Us"), em que
"*"
significa "vezes". O quadrado, que era de X²"Us quadrados" agora vai ser
(X²/4)*("4 Us quadrados" = "4Us"); ou seja, o número que expressa o
comprimento é mudado mais o comprimento permanece o mesmo (medido em cm).
Isso é compreensível. Mas, no algorítmo da  RPM, se trocamos o segmeno
unitário por um outro 2 vezes maior (ou seja, trocamos U por 2U),
quadrado
do segmento X  fica 2 vezes menor (em cm)para que o triângulo não deixe
de
ser retângulo. Medindo em 2U, X fica 2 vezes menor e seu quadrado fica 4
vezes menor. OK. Mas aparece um problema: O tamanho do quadrado de X fica
diferente, mesmo medindo em centímetros!! Isso não é estranho? A troca de
unidades influi no tamanho REAL de um segmento? Ou seja, não posso
desenhar
dois segmentos e dizer que um é o quadrado do outro, porque isso só é
verdade para uma certa unidade de medida?Estou confuso!!!
Espero que respondam meu e-mail; mas obrigado só pela atenção
[]s, J.P. (que honra ter essas iniciais...)
_
Get Your Private, Free E-mail from MSN Hotmail at http://www.hotmail.com.

Share information about yourself, create your own public profile at
http://profiles.msn.com.








Re: estranho

2000-09-09 Por tôpico José Paulo Carneiro




Josimat:
A sua 
estranheza vem do fato de que frequentemente se ouve ou le que a definicao de 
probabilidade eh o quociente entre o numero de casos favoraveis e o numero de 
casos possiveis. Se isto fosse verdade, voce teria razao: probabilidade zero 
seria sinonimo de evento impossivel. Acontece que esta nao eh a definicao de 
probabilidade de um evento. Isto eh aquilo a que se reduz a definicao quando se 
particulariza para conjuntos (espacos amostrais) finitos (e mesmo assim, supondo 
certas coisas).
De um 
modo mais geral (e mesmo assim ainda nao eh o super-geral), a probabilidade eh o 
quociente entre uma certa medida do conjunto dos casos favoraveis e 
a medida do conjunto dos casos possiveis. So para conjuntos finitos, 
esta medida eh o numero de elementos do conjunto em questao.
Para 
conjuntos infinitos, isto seria impossivel. Para estes, entao, sao introduzidas 
outras maneiras de medir esses conjuntos (por exemplo, comprimentos, areas, 
etc.), e ahi, podem existir conjuntos de medida zero que nao sao 
vazios.
JP

-Mensagem original-De: 
josimat [EMAIL PROTECTED]Para: 
[EMAIL PROTECTED] [EMAIL PROTECTED]Data: 
Sexta-feira, 8 de Setembro de 2000 20:08Assunto: 
estranho
Muito 
estranho...
Algum poderia dizer algo sobre a 
afirmativa:
A 
probabilidade de um evento ocorrer pode ser zero mesmo sendo possvel sua 
ocorrncia.
[]'s 
Josimar



Re: P.G.

2000-09-02 Por tôpico José Paulo Carneiro

A primeira progressao tem razao igual a raiz de 2, que eh maior que 1.
Logo a sua "soma infinita" nao faz sentido.
JP
-Mensagem original-
De: {O-Grande-Mentecapto} [EMAIL PROTECTED]
Para: [EMAIL PROTECTED] [EMAIL PROTECTED]
Data: Sexta-feira, 1 de Setembro de 2000 23:41
Assunto: P.G.


Oi.. algúem pode dar uma luz? :)

149. Calcule, quando existir, a soma dos termos da P.G. infinita que está
dada:

a) (sqrt(2)/4, 1/2, sqrt(2)/2, ...)
b) (sqrt(2)/2, 1/2, sqrt(2)/4, ...)

para o a:
q =  2/sqrt(2) simplificando q = sqrt(2)

para o b:
q = sqrt(2)/2 q = sqrt(2)/2

substituindo o q e a1 do exercício b na fórmula de soma da pg infinita
(a1/q -1) temos: S = 1 + sqrt(2) como resposta.
agora o estranho.. fazendo o mesmo processo com o exercício a, o resultado
sempre fica com uma raiz no denominador e fica impossível de racionalizar o
resultado.

na resposta do a) eles colocam um simbolo que eu nunca vi antes.. parece um
A em italico mais uns tracinhos puxados.

* imagino que deva ser por causa da não racionalização da soma do
segundo... , mas porque isso só ocorre nele se as P.G. são iguais, só
'inversas'?



"Against stupidity, the Gods themselves contend in vain",
 Friedrich von Schiller's
-
[]'s
{O-Grande-Mentecapto}
[EMAIL PROTECTED]







Re: Possível injustiça

2000-09-02 Por tôpico José Paulo Carneiro

Gostaria de acrescentar as observacoes do Nicolau o seguinte:
Todos os professores que trabalham nas Olimpiadas de Matematica nao ganham
nenhum dinheiro com isto. Todos os alunos que participam das Olimpiadas nao
ganham premios em dinheiro, apenas medalhas e diplomas. Alem disto, "perdem"
seu tempo de sabados, domingos e feriados para tentar resolver problemas
considerados dificeis ou impossiveis pela maioria, inclusive por nos,
professores
(quando  sao formulados por outros colegas). Ainda assim, so no Rio de
Janeiro,
mais de 3000 criancas e jovens participaram da 1a fase da OBM em 2000.
Trata-se de um fenomeno educacional impressionante, e que deveria constituir
um ponto de reflexao para todos nos, professores de Matematica. O que estao
esses jovens procurando na Olimpiada, e que certamente o ensino "normal" nao
lhes oferece?
Seria preciso alguem muito singular (para nao dizer cretino) para querer
fraudar um sistema como este, que nao envolve dinheiro. Tal fraudador
deveria empregar de modo mais "util" suas "habilidades" a este respeito. Mas
mesmo que isto ocorresse (e ja ouvi falar de um caso a nivel internacional),
as pessoas conhecedoras do sistema logo detectariam, principalmente se isto
ocorresse de forma sistematica, de forma a beneficiar sempre os mesmos (o
criminoso sempre volta ao local do crime). Em todo caso, sua duvida eh
procedente, e estah sempre na cabeca dos organizadores, que procuram tomar
os cuidados plausiveis.
Jose Paulo

-Mensagem original-
De: Nicolau C. Saldanha [EMAIL PROTECTED]
Para: [EMAIL PROTECTED] [EMAIL PROTECTED]
Data: Sábado, 2 de Setembro de 2000 14:30
Assunto: Re: Possível injustiça




On Fri, 1 Sep 2000, Jorge Peixoto  Morais wrote:

 Como é o sistema de pontuação da OBM nível 2? Eu não me lembro direito,
mas
 acho que a 1ª fase vale 20 pontos, a 2ª 40 e a 3ª 300. E as duas
primeiras
 não só servem para eliminar, como também contam seus pontos na
classificação
 final. Se assim for, o que impede alguém de competir desonestamente, já
que
 as duas primeiras fases são realizadas na escola?


Nós temos por princípio confiar na honestidade dos participantes e das
escolas. Seria possível que uma escola favorecesse indevidamente
seus estudantes sim, mas não acreditamos que isto ocorra freqüentemente.
Note que o peso da terceira fase é muito maior exatamente
para que eventuais desonestidades tenham um efeito mínimo.
[]s, N.






Re: Permutações caóticas

2000-09-02 Por tôpico José Paulo Carneiro




Leia o meu artigo O problema do 
amigo oculto, na revista do Professor 
de Matematica, no 28, que eh 
exatamente sobre isto.
JP

-Mensagem original-De: 
Franklin de Lima Marquezino [EMAIL PROTECTED]Para: 
[EMAIL PROTECTED] [EMAIL PROTECTED]Data: 
Sbado, 2 de Setembro de 2000 09:08Assunto: 
Permutaes caticas
Ol,

 H algum 
tempo eu perguntei nesta lista, como se calcula permutaes 
caticas, e ningum me respondeu at hoje. Estou no 3 
ano do ensino mdio, e sei que minha pergunta pode ter sido um pouco 
idiota. Porm, eu j havia comentado esta minha dvida com 
alguns de meus amigos, que tambm participam desta lista, e nenhum deles 
soube responder. Ento, como vocs dizem que aqui predomina a 
solidariedade e a cooperao, eu pensei que no houvesse 
problema em compartilhar minha dvida, por mais simples que 
fosse.
Caso algum tenha tempo, por favor, 
responda.


 At 
logo,


 
Franklin



Re: Análise Combinátoria

2000-08-30 Por tôpico José Paulo Carneiro

Imagine primeiro todos os anagramas sem esta restricao, isto eh,
todas as permutacoes das cinco letras, em numero de 5!=120.
Se voce fixar uma certa arrumacao das consoantes, e permutar as vogais,
das quais existem 3!=6 permutacoes, vai ver que dessas 6 possibilidades,
so uma tem as vogais em ordem alfabetica. Como isto vale para cada
arrumacao das consoantes, o numero que voce procura eh:
5!/3!= 120/6=20.
Outro racicocinio eh o seguinte: Escolha 3 lugares para colocar as vogais.
Isto pode ser feito de C(5;3)=5x4x3/1x2x3= 10 maneiras. Para manter a ordem
alfabetica, nada mais ha a fazer com as vogais. Agora, para cada uma dessas
10 escolhas, permute as 2 consoantes, obtendo 20.
A proposito, se voce fizer isto literalmente, voce terah provado que:
C(m;p) x (m-p)! = m! / p!, ou seja:
C(m;p) = m! / (p! (m-p)!)
Ou seja, isto eh uma deducao da formula das combinacoes!
Veja no Eureka 6 o artigo "Contar duas vezes para generalizar"...
Jose Paulo


-Mensagem original-
De: João Paulo Paterniani da Silva [EMAIL PROTECTED]
Para: [EMAIL PROTECTED] [EMAIL PROTECTED]
Data: Terça-feira, 29 de Agosto de 2000 23:40
Assunto: Análise Combinátoria



   Olá. Estou na segunda série do Ensino Médio.

  Quantos anagramas da palavra "aluno" têm as vogais em ordem alfabética?

João Paulo Paterniani da Silva

_
Get Your Private, Free E-mail from MSN Hotmail at http://www.hotmail.com.

Share information about yourself, create your own public profile at
http://profiles.msn.com.







Re: AjudA!! Complexos (geometria)

2000-08-25 Por tôpico José Paulo Carneiro

Observe que a, b, c sao colineares se e so se c-a eh um multiplo real
se b-a, ou seja, (c-a)/(b-a) eh real, ou ainda este quociente eh igual ao
 seu conjugado. Agora, manipule algebricamente as 4 igualdades resultantes,
use as propriedades tipicas do conjugado, para chegar a uma outra igualdade
envolvendo so a', b'e c'. (isto eh, elimine a, b, c e seus conjugados).
Tenha fe e coragem que acaba saindo.
JP

-Mensagem original-
De: Marcio [EMAIL PROTECTED]
Para: [EMAIL PROTECTED] [EMAIL PROTECTED]
Data: Sexta-feira, 25 de Agosto de 2000 00:17
Assunto: AjudA!! Complexos (geometria)


 Alguem consegue resolver esse problema??

Sejam a;b;c;a';b';c' numeros complexos.
Sabe-se que (a;b;c') sao colineares, assim como (a';b;c), (a;b';c) e (a'/a;
b'/b; c'/c; 1).

Mostre que os complexos a';b';c' sao colineares.

[]'s
Marcio






Re: Olimpíada Estadual do Rio de Janeiro

2000-08-23 Por tôpico José Paulo Carneiro




Eh claro que estao, ja que a primeira 
prova foi comum.
Apenas 
a nota de corte foi 1 ponto mais baixa, em cada nivel.
JP

-Mensagem original-De: 
Marcos Paulo [EMAIL PROTECTED]Para: 
[EMAIL PROTECTED] [EMAIL PROTECTED]Data: 
Quarta-feira, 23 de Agosto de 2000 21:24Assunto: Olimpada 
Estadual do Rio de Janeiro
Gostaria de saber seos colgios 
inscritos na OBM esto automaticamente inscritos na estadual (colegios do 
Rio de Janeiro claro)
Obrigado.

[]'s MP


Re: Re: Re: sugestão

2000-08-19 Por tôpico José Paulo Carneiro

Eh verdade. Tambem estiveram no Brasil o Dieudonne (a alma do Bourbaki)
e o Grothendieck.
JP

-Mensagem original-
De: [EMAIL PROTECTED] [EMAIL PROTECTED]
Para: [EMAIL PROTECTED] [EMAIL PROTECTED]
Data: Sexta-feira, 18 de Agosto de 2000 18:11
Assunto: Re: Re: Re: sugestão


Obrigado ao JP pela correção, eu acabei criando um terceiro matemático
(uma mistura de André Veil com Andrew Wiles, lamentável confusão.)

A próposito, Andre Veil não esteve desenvolvendo alguns trabalhos por aqui
(acho que antes de 1950) mais ou menos na epóca de grande atividade de
Leopoldo Nachbin (um dos fundadores do IMPA). Se não me engano li algo
sobre isso num pequeno livreto, uma espécie de homenagem que seu filho (Um
pesquisador do IMPA) fez à esse grande lutador pela matemática no brasil
que, como de costume, não recebe o valor merecido.

Estou enganado JP ? ou quem saiba...

[]'s
Alexandre Vellasquez



Mais uma vez concordo com o Alexandre.

Esclarecimento historico:
Andrew Wiles: matematico que demonstrou o grande teorema de Fermat.
Andre Veil: matematico frances que fez parte do grupo Bourbaki.
Hermann Weyl: matematico alemao, um dos ultimos da escola de Goettingen,
que acabou desbaratada a partir de 1933, com a ascensao do nazismo.
JP







Re: Re: sugestão

2000-08-18 Por tôpico José Paulo Carneiro

Mais uma vez concordo com o Alexandre.

Esclarecimento historico:
Andrew Wiles: matematico que demonstrou o grande teorema de Fermat.
Andre Veil: matematico frances que fez parte do grupo Bourbaki.
Hermann Weyl: matematico alemao, um dos ultimos da escola de Goettingen,
que acabou desbaratada a partir de 1933, com a ascensao do nazismo.
JP

-Mensagem original-
De: [EMAIL PROTECTED] [EMAIL PROTECTED]
Para: [EMAIL PROTECTED] [EMAIL PROTECTED]
Data: Sexta-feira, 18 de Agosto de 2000 09:54
Assunto: Re: Re: sugestão


Estive pensando (ihhh...) um pouco sobre esse negócio de questões triviais
e questões difíceis, senão vejamos:

Eu encaro da seguinte forma:  Não há questão, qualquer que seja o seu
nível, que não mereça atenção de alunos ou professores. Se um leitor a
considerar elementar para o seu nível de conhecimento ele deve se lembrar
que ao emitir comentários, sugerir leituras sobre o assunto ou mesmo
mostrar o caminho da solução, ele estará ajudando: 1)à quem enviou à
dúvida; 2)estará ajudando a tantos outros que talvez ainda não tenham
percebido que possuem a mesma dúvida; 3) por último estará no mínimo
exercitando, mesmo que sem perceber, os seus conhecimentos.
Caso a questão seja de nível superior ao conhecimento de um leitor, isso
deve servir de incentivo, motivação, para que ele procure aprender um
pouco mais, mesmo que gradualmente, sobre os assuntos relacionados à
questão.

Vocês já pensaram se o Ralph , O Gugu , o Nicolau  (até aqui já são 4
medalhas de ouro em IMO's), O Eduardo Wagner, O Morgado, e tantos outros,
simplemente resolvessem pensar " Ahh, essa questão é trivial demais para o
meu nível... vou ignorá-la" quando encontrassem questões que para eles
fossem elementares, mas que para nós não são??  O que seria dessa nossa
discussão em matemática?.

Por outro lado, Andrew Weil (é assim??) conheceu o Teorema mais famoso do
mundo (lembram, o ùltimo de Fermat.) com apenas 10 anos de idade, e o
perseguiu a vida toda. Ou seja, ele desenvolveu matemática exatamente por
ter se defrontado com um problema que ele (e todo mundo, literalmente!)
simplesmente não consegui resolver. E se ele tivesse, como tantos fizeram,
desistido por pensar " Ahhh.. isso está bem acima do meu nível!". Mas não,
ele preferiu encara de outra forma,aprendeu matemática gradualmente e isso
era apenas o começo da brincadeira.

Eu quando era do primário ficava fascinado com o pessoal do ginásio porque
eles somavam x e y e encontram números como resposta. Eu ainda não sabia
nada sobre o assunto, e pra mim aquilo parecia difícil. Acho que foi por
isso que resolvi estudar matemática...

Era isso... desculpem o tempo tomado!

[]'s,
Alexandre Vellasquez



se alguém acha um problema muito difícil, pode simplesmente ignorá-lo; se
o acha trivial, pode descartá-lo da mesma forma. E se alguém quiser propor
um problema para as duas listas? Então todos aqueles que se inscreverem
nas duas listas receberão uma mensagem em duplicata.

Essa é a minha opinião.







Re: Permutação Caótica

2000-08-16 Por tôpico José Paulo Carneiro




A respeito de permutacoes caoticas, 
menciono o meu artigo
O problema do amigo oculto, na Revista do Professor de 
Matematica,
no. 28, p.21 (atencao! la nao ha os 
elementos repetidos).
JP

-Mensagem original-De: 
Marcos Paulo [EMAIL PROTECTED]Para: 
[EMAIL PROTECTED] [EMAIL PROTECTED]Data: 
Quarta-feira, 16 de Agosto de 2000 19:15Assunto: Re: 
Permutao Catica


Como se calcula o nmero de 
permutaes caticas em um conjunto com elementos 
repetidos?

Uma pergunta equivalente a essa que talvez 
fique mais clara seria ( se entendi bem) quantos so os anagramas da 
palavra MATEMATICA em que nenhuma das letras ocupa a posico ocupada 
na palavra MATEMATICA
acho que eh isso ... 
quanto a soluo 


[]'s MP


Re: argumentos combinatórios

2000-07-30 Por tôpico José Paulo Carneiro

Isto estah feito em um artigo meu no Eureka, chamado:
"Contar duas vezes, para generalizar". Acho que eh o no 6.
JP

-Mensagem original-
De: Marcelo Souza [EMAIL PROTECTED]
Para: [EMAIL PROTECTED] [EMAIL PROTECTED]
Data: Domingo, 30 de Julho de 2000 17:19
Assunto: argumentos combinatórios


Fala, galera!

 Alguém poderia resolver este problema pra mim?
- Diga, utilizando argumentos combinatórios o valor de

( n )^2( n )^2 ( n )^2
(   )   +  (   )   + ... + (   )
( 0 )  ( 1 )   ( n )

Obrigado
Abraços
Marcelo
P.s.
( n )
(   ) = n!/k!(n-k)!
( k )

Get Your Private, Free E-mail from MSN Hotmail at http://www.hotmail.com














Re: i^i ; Moebius....

2000-07-15 Por tôpico José Paulo Carneiro


-Mensagem original-
De: Bruno Woltzenlogel Paleo [EMAIL PROTECTED]
Para: Lista OBM [EMAIL PROTECTED]
Data: Quinta-feira, 13 de Julho de 2000 20:08
Assunto: i^i ; Moebius



Quando eu mandei a HP calcular, ela retornou um par ordenado, onde um dos
"elementos" era o número que eu encontrei e o outro era o Zero...

= O complexo a+bi pode ser identificado com o par ordenado (a;b). Nesta
identificacao, o real x eh: (x;0).




Re: faixa de hipérbole

2000-07-09 Por tôpico José Paulo Carneiro

Seja x_0 = a  x_1  ...  x_n = b a particao do inetrvalo.
Da figura, tira-se que a area sob a hiperbole menos a area do retangulo
inscrito (tudo isto no k-esimo intervalo) eh menor que a do triangulo
retangulo
de catetos x_k - x_(k-1) e F(x_(k-1))-F(x_k), ou seja:
(1/2)(x_k-x_(k-1)*(1/x_(k-1)- 1/x_k), a qual eh menor que :
(1/2) c * (1/x_(k-1)- 1/x_k).
Somando estas desigualdades para k de 1 a n, tem-se que a diferenca
entre a area da faixa de hiperbole e a soma das areas dos retangulos
inscritos eh menor que:  (1/2) c * (1/x_0 - 1x_n)  = (1/2) c * (1/a -1/b),
ja que
a 1a parcela -1/x_1 cancela com a seguinte 1/x_1, etc.
JP



-Mensagem original-
De: Marcelo Souza [EMAIL PROTECTED]
Para: [EMAIL PROTECTED] [EMAIL PROTECTED]
Data: Domingo, 9 de Julho de 2000 10:59
Assunto: faixa de hipérbole


Olá
Alguém poderia me ajudar enviando a solução do exercício abaixo
- Dada a decomposição di intervalo [a,b] em intervalos justapostos, o
"erro"
que se comete ao tomar-se a área do polígono retangular P ao invés da área
da faixa H(a)_b é a diferença E=(área de H(a)_b) - (área de P). Prove que
se
tem Ec/2(1/a-1/b), onde c é o comprimento do maior intervalo da
decomposição.
Conclua que, fixado [a,b], podemos tornar o erro E tão pequeno quanto se
deseje (digamos Ee') desde que tomemos uma decomposição de [a,b] por meio
de intervalos de pequeno comprimento. (digamos, todos menores que a.e'). em
particular, o erro que se comete ao se substituir a área da faixa H(b)_1
pela área de um polígono retangular inscrito é inferior ao comprimento do
maior intervalo da decomposição.
Notações:
H(a)_b = faixa da hiperbole limitada pelas abscissas a e b, ba
não sei se precisa dizer, mas a função que determina essa hipérbole é
y=1/x...
Obrigado
Abraços
Marcelo

Get Your Private, Free E-mail from MSN Hotmail at http://www.hotmail.com





Re: uma desigualdade!

2000-07-09 Por tôpico José Paulo Carneiro

Este problema pode ser resolvido de modo analogo ao da hiperbole:
A soma 1/2^3 + 1/3^3 + ... + 1/n^3 eh a soma das areas dos
retangulos inscritos sob a curva y=1/x^3, de 1 ateh n, para a particao:
123...n. Entao, ela eh menor que a integral de 1/x^3 dx de 1 a n, a qual
eh:
1/2 - 1/2n^2  1/2. Somando 1 a ambos os lados, a soma he menor que 3/2.
JP


-Mensagem original-
De: Carlos Gomes [EMAIL PROTECTED]
Para: [EMAIL PROTECTED] [EMAIL PROTECTED]
Data: Domingo, 9 de Julho de 2000 22:14
Assunto: uma desigualdade!


Caros amigos, como posso verificar a desigualdade
  1/1^3  + 1/2^3  + 1/3^3  + ...+ 1/n^3 3/2   para todo n natural ?

Um abraço ,
Carlos A Gomes.







Re: apreciação

2000-07-08 Por tôpico José Paulo Carneiro

Esta me pareceu a melhor solucao ateh agora do problema.
So 2 comentarios:
1) espero que todos tenham entendido que (a,b) estah
significando o m.d.c. de a e b (eu nao gosto desta notacao,
embora seja bastante usual).
2) Quando diz: "agora basta ver o que ocorre para (A,B)=1", pode
dar a impressao de que esse caso vai esclarecer algum mais geral;
na verdade: (A,B)=1 mesmo.
JP

-Mensagem original-
De: Ecass Dodebel [EMAIL PROTECTED]
Para: [EMAIL PROTECTED] [EMAIL PROTECTED]
Data: Sábado, 8 de Julho de 2000 16:38
Assunto: Re: apreciação




Eu mesmo tenho dificuldade de ler os sinais das minhas mensagens. Vou
substituir o sinal de mais por #, assim talvez todos consigam ler:

Seja (a,b)=g, e a=gA, e b=gB
(a^2 # b^2)/ab = (A^2 # B^2)/AB

Agora basta ver o que ocorre para (A,B)=1. Mas veja que
- se (A,B)=1 então (A # B,B)=(A # B,A)=1
- se (A # B,B)=(A # B,A)=1 então (A # B,AB)=1
- se (A # B,AB)=1 então ((A # B)^2,AB)=1
Logo (A # B)^2/AB = (A^2 # B^2)/AB + 2 é inteiro somente se AB=1,
portanto
A=1 e B=1, logo a=b=g.

Tudo perfeitamente certo?

Meus comentários: na solução do amigo Marcos Eike Tinen dos Santos, me
PARECE que ele admite algo como: para (a # b)/c ser inteiro a/c e b/c
precisam ser inteiros, o que não é verdade. (1 # 3)/2 é inteiro.
Na solução do amigo Filho, que me PARECE perfeita, há o uso da fórmula das
raízes da equação de segundo grau, o que, apesar de não ser tão artificial,
já não é tão "simples" quanto à solução que eu dou acima.

Obrigado!

Eduardo Casagrande Stabel.


Get Your Private, Free E-mail from MSN Hotmail at http://www.hotmail.com





Re: ajuda

2000-07-05 Por tôpico José Paulo Carneiro





-Mensagem original-De: 
Andr Amiune [EMAIL PROTECTED]Para: 
[EMAIL PROTECTED] [EMAIL PROTECTED]Data: 
Tera-feira, 4 de Julho de 2000 23:34Assunto: Re: 
ajuda
1. Acho uma prova poderia ser assim:

a^2+b^2 = (a+b)^2 -2ab, logo para ab dividir (a^2 + 
b^2), ab deve dividir a+b.

=Nao concordo. isto prova apenas que ab divide 
(a+b)^2. 
9 divide 6^2, mas nao divide 6.

Se considerarmos b = a+ k temos que provar que 
a(a+k) divide a + (a+k) = 2a + k somente se k=0.

Uma das condies para 2a + k 
ser divisvel por a(a+k)  que 2a + k seja mltiplo de a e 
logo k' = a.k.
Substituindo:

a^2(k' +1) deve dividir a(k'+2) 

Alm de k'+2 ter queser 
mltiplo de a, uma das condies necessrias para 
isso ser verdade que que jamias severificar para k  
0  k'+1 dividir k'+2, pois ambos so primos entre si. 
Logo a^2(k'+1) s divide a(k'+2) se k'=k=0 e temos a=b.

Certo?


- Original Message - 
From: 
Filho 

To: discusso de problemas 
Sent: Tuesday, July 04, 2000 10:49 
AM
Subject: ajuda

1.Sejam a e b inteiros positivos. Se a^2 + b^2  
divisvel por ab, mostre que a=b.

2. Demonstrar que a equao x^2 + y^2 - z^2 
= 1997 tem infinitas solues inteiras.


Grato


Re: sem cálculo

2000-07-01 Por tôpico José Paulo Carneiro




A afirmacao existe uma e so 
uma pode ser decomposta em 2 partes:
existe no maximo uma e existe no minimo 
uma.

A primeira pode ser decidida por argumentos que nao 
envolvem analise;
por exemplo, se a e b sao raizes , entao 
a^3-b^3=2(b-a) ou a^2+ab+b^2=2, 
donde se conclui que ab= 0 (ja que a e b estao 
entre -1 e 1), etc. 

Mas a afirmativa de existencia sempre 
vai exigir algum argumento de analise
do 
tipo se um polinomio eh positivo para x=c e negativo para x=d , entao tem 

uma raiz entre c e d (citado abaixo). Na 
realidade, a propria existencia da raiz cubica 
(e da raiz quadrada) de um real qualquer 
envolve argumentos deste tipo. A questao eh que 
esta existencia (e outras) entra sorrateiramente no ensino medio, sem justificativa. 


Agora, em relacao a derivada, concordo. Muita coisa 
(talvez tudo) que se faz com derivadas
para polinomios, pode ser feito algebricamente (a 
formula de Taylor eh exata para
polinomios). 
Por exemplo, esqueca que a derivada eh 3x^2+2. 
Examinar se f(x)=x^3+2x+k eh crescente
equivale a examinar se 
f(b)-f(a)=(b-a)(b^2+ab+^2+2) tem o mesmo sinal de b-a, ou 
seja,
se eh positivo o ultimo parenteses, o qual = 
(a+(b/2))^2+(3b^2)/4 + 2.
JP
(PS: quando ja ia enviar este mail, 
vi que o Ralph escreveu algo analogo)
JP

-Mensagem original-De: 
Filho [EMAIL PROTECTED]Para: 
discusso de problemas [EMAIL PROTECTED]Data: 
Sbado, 1 de Julho de 2000 14:07Assunto: sem 
clculo
Caro Wellington no final do seu 
comentrio, voc usou recursos de clculo. A questo 
foi de um vestibular que no programa no consta nada de 
clculo.
Grato pelo primeiro comentrio, mas o que 
torna a questo diferente  exatamente no poder usar tais 
recursos. O problema continua

Mostre que a equao x^3 + 2x 
+k=0, com k real no intervalo aberto ]-3,3[, possui exatamente uma raiz no 
intervalo aberto ]-1,1[.

Seja f(x)=x^3+2x+k;Primeiramente substituiremos x nos valores extremos 
do intervalo:para x=-1 a imagem da funcao estara em ]-6,0[;para x=1 a 
imagem da funcao estara em ]0,6[;ou seja, independente do valor de k dentro 
do intervalo em questao ( ]-3,3[ ), a funcao retornara valores com sinais 
opostos. Isso garante a existencia de um numero impar de razes nesse 
intervalo (Teorema de Bolzano). (Para que exista apenas uma raiz, a 
funcao nesse caso deve ser estritamente crescente. Analisaremos entao a sua 
derivada:f ' (x)= 3x^2+2  0 para todo x, o que termina o 
problema)?? Pensem conosco, 
grato


Re: área

2000-06-29 Por tôpico José Paulo Carneiro




Eh claro que nao. Pegue um descanso 
de prato desses que encolhem e esticam.
Com o 
mesmo perimetro, voce obtem areas diferentes.

-Mensagem original-De: 
Aron Roberto Ferreira [EMAIL PROTECTED]Para: [EMAIL PROTECTED] [EMAIL PROTECTED]Data: 
Quinta-feira, 29 de Junho de 2000 22:07Assunto: 
rea
Ol pessoal
 Gostaria de saber se de alguma maneira 
 possvel determinar a rea de um quadriltero 
conhecendo apenas as medidas de seus lados (mantendo o permetro, 
lgico)?.
Obrigado!

 Edmilson, realmente 
voc estava certo, nem sempre a recproca  
verdadeira.


Re: ajuda

2000-06-19 Por tôpico José Paulo Carneiro




Uma parabola.
Observe que a eq. pode ser escrita: 
(x-2y)^2-3x-y-1=0.
A reta 
x-2y=0 forma com o eixo X um angulo alfa tal que
cos(alfa) =2/(raiz de 5) e sen(alfa)=1/(raiz de 5).
Se voce fizer entao uma mudanca de 
variaveis
(2x+y)/(raiz de 5)=u
(-x+2y)/(raiz de 5)=v
(equivalente a uma rotacao de (-alfa) dos eixos em 
torno da origem),
voce chegarah a uma equacao do tipo v = au^2 + bu + 
c,
onde reconhecerah uma parabola, da qual voce 
poderah calcular
foco, parametro, etc,
JP


-Mensagem original-De: 
Filho [EMAIL PROTECTED]Para: 
discusso de problemas [EMAIL PROTECTED]Data: 
Segunda-feira, 19 de Junho de 2000 09:52Assunto: 
ajuda
Qual  a cnica representada pela 
equao x^2 + 4y^2 -4xy - 3x - y - 1=0 ?



obs: x^2 representa x elevado a 2


Re: soluções inteiras

2000-06-16 Por tôpico José Paulo Carneiro

Aqui tambem! Mas eu nao abri.
JP

-Mensagem original-
De: Mira [EMAIL PROTECTED]
Para: [EMAIL PROTECTED] [EMAIL PROTECTED]
Data: Sexta-feira, 16 de Junho de 2000 13:52
Assunto: Re: soluções inteiras


Cuidado Carlos, sua mensagem chegou por aqui com o Happy99.exe atachado!

- Original Message -
From: Carlos Gomes [EMAIL PROTECTED]
To: [EMAIL PROTECTED]
Sent: Thursday, June 15, 2000 11:31 PM
Subject: soluções inteiras


Alô pessoal como estão todos, tudo ok?

Gostaria que alguém me ajudasse com s seguinte questão que
 está no livro teoria elementar dos números do Edgar de Alencar, logo no
 primeiro capítulo:

  Achar todas as soluções inteiras e positivas da equação
 (x+1)(y+2)=2.x.y

 Um abraço a todos,

 Carlos A. Gomes
   15.06.00








Re: Re: como achar?

2000-06-05 Por tôpico José Paulo Carneiro

Estah correto o seu teorema. So que a condicao de que a derivada
seja continua eh muito forte. Basta que ela exista.

Outra coisa: nao gosto desta definicao de continuidade, que voce diz que eh
do Aurelio (nao eh um bom lugar para definicoes matematicas: veja o
comentario
em "Matematica do Ensino medio", Vol.I, de Elon Lima e outros, sobre a
definicao de numero).  Esta definicao serve para o seu caso, em que o
dominio eh um intervalo, mas nao para o caso geral em que o dominio pode ter
pontos isolados, onde a funcao estah definida no ponto, eh continua, e o
limite nao existe. Uma funcao eh continua em c quando x (do dominio) proximo
de c implica f(x) proximo de f(c), e isto pode ser traduzido
(equivalentemente) por vizinhancas (epsilons e deltas) ou por sequencias.

JP

-Mensagem original-
De: Ecass Dodebel [EMAIL PROTECTED]
Para: [EMAIL PROTECTED] [EMAIL PROTECTED]
Data: Domingo, 4 de Junho de 2000 19:11
Assunto: Re: Re: como achar?





From: "José Paulo Carneiro" [EMAIL PROTECTED]
Reply-To: [EMAIL PROTECTED]
To: [EMAIL PROTECTED]
Subject: Re: como achar?
Date: Sun, 4 Jun 2000 09:39:04 -0300

Dois comentarios:
1) mais uma vez, recomendo a leitura de dois artigos da RPM:
um do Wagner: "Os numeros a^b e b^a" (RPM 28), e outro meu:
"Voltando aos numeros a^b e b^a" (RPM 31).
2) Lembro que a nulidade da derivada em um ponto nao eh condicao
suficiente para ocorrencia do maior valor de uma funcao, como parecem
sugerir certas resolucoes que terminam por ahi.
JP


Eu havia dito em meu e-mail sobre o problema do Benjamin:

" tome y(x)=x^(1/x), e veja que se a funcao tem um maximo, eh onde a
derivada e' nula (nesse caso): "

Nao sei o que deu a entender; mas eu quis dizer exatamente o que o Jose
Paulo Carneiro falou, isto é, existem casos onde o maximo de uma função não
é onde a derivada é nula. Peço perdão se não deixei isso claro.

Vou dar um exemplo bem simples:
Suponha que y(x)=x, e queremos achar o valor máximo dessa função com x no
intervalo [0,5]. É claro que o máximo é em x=5, mas a derivada é 1 nesse
ponto.

Vou tentar dar uma condição para que a função y(x), com x num intervalo I
(contínuo, sem quebras), tenha seu máximo onde a derivada é nula:

i. suponhamos que y(x) é contínua em I (por função contínua dizemos: a)
para
todo x E I, y(x) está definida; b) o limite da função no ponto coincide com
o valor da função no ponto; do dicionário Aurélio), equivale a dizer
intuitivamente que a função não dá saltos.
ii. suponhamos que y(x) não dê viradas bruscas, em outras palavras, que a
sua derivada seja contínua no intervalo I.
iii. suponhamos que y(x) tenha um valor máximo em I, e ele não seja nenhum
dos extremos de I (menor e maior valores)

Vou me arriscar a dizer que se uma função obedece a i, ii e iii então o
máximo de y(x), com x E I, vai ser em algum dos pontos onde a derivada for
nula. Suponhamos {x1,x2,...,xn} todos os pontos onde
y'(x1)=y'(x2)=...y'(xn)=0, então o maior valor de {y(x1),y(x2),...,y(xn)}
será o máximo da função y(x) no intervalo I.

Algum comentário? Algo mais esclarecedor?

Eduardo Casagrande Stabel.



Get Your Private, Free E-mail from MSN Hotmail at http://www.hotmail.com





Re: Re: como achar?

2000-06-05 Por tôpico José Paulo Carneiro

Estah perfeitamente esclarecido.
JP

-Mensagem original-
De: Alexandre Gomes [EMAIL PROTECTED]
Para: [EMAIL PROTECTED] [EMAIL PROTECTED]
Data: Domingo, 4 de Junho de 2000 21:57
Assunto: Re: Re: como achar?





From: "Ecass Dodebel" [EMAIL PROTECTED]
Reply-To: [EMAIL PROTECTED]
To: [EMAIL PROTECTED]
Subject: Re: Re: como achar?
Date: Sun, 04 Jun 2000 22:02:49 GMT




From: "José Paulo Carneiro" [EMAIL PROTECTED]
Reply-To: [EMAIL PROTECTED]
To: [EMAIL PROTECTED]
Subject: Re: como achar?
Date: Sun, 4 Jun 2000 09:39:04 -0300

Dois comentarios:
1) mais uma vez, recomendo a leitura de dois artigos da RPM:
um do Wagner: "Os numeros a^b e b^a" (RPM 28), e outro meu:
"Voltando aos numeros a^b e b^a" (RPM 31).
2) Lembro que a nulidade da derivada em um ponto nao eh condicao
suficiente para ocorrencia do maior valor de uma funcao, como parecem
sugerir certas resolucoes que terminam por ahi.
JP

   Talvez um dos interessados diretos por esta resposta seja eu, uma vez
que
utilizei o fato acima descrito para resolver a questão. Sei que se a
derivada primeira de uma função for nula em um ponto, este fato não é
condição suficiente para a ocorrência de máximo. O que eu fiz foi APENAS
tentar chegar até a mesma solução do livro do Benjamin, sem querer entrar
em
um estudo mais detalhado da função(não sei se esta foi a intenção do
remetente), já que este alegou não ter acertado a derivada da função(talvez
por isso não fosse interessante tentar ir além disso, pois dá a entender
que
o remetente não tem um curso de cálculo). Então eu parei neste ponto.
   Reconheço que posso ter errado em induzir um conceito errado em um
componente da lista, mas esta não foi a minha intenção.
   Atenciosamente
Alexandre S. Gomes.
___
Get Your Private, Free E-mail from MSN Hotmail at http://www.hotmail.com



Get Your Private, Free E-mail from MSN Hotmail at http://www.hotmail.com





Re: como achar?

2000-06-04 Por tôpico José Paulo Carneiro

Dois comentarios:
1) mais uma vez, recomendo a leitura de dois artigos da RPM:
um do Wagner: "Os numeros a^b e b^a" (RPM 28), e outro meu:
"Voltando aos numeros a^b e b^a" (RPM 31).
2) Lembro que a nulidade da derivada em um ponto nao eh condicao
suficiente para ocorrencia do maior valor de uma funcao, como parecem
sugerir certas resolucoes que terminam por ahi.
JP

-Mensagem original-
De: Alexandre Gomes [EMAIL PROTECTED]
Para: [EMAIL PROTECTED] [EMAIL PROTECTED]
Data: Sábado, 3 de Junho de 2000 18:41
Assunto: Re: como achar?





From: "Benjamin Hinrichs" [EMAIL PROTECTED]
Reply-To: [EMAIL PROTECTED]
To: "Obm-l" [EMAIL PROTECTED]
Subject: como achar?
Date: Sat, 3 Jun 2000 17:13:54 -0300

Pessoal,
perguntinha para vós: como se acha o valor máximo de x^(1/x). Como se
chega
ao valor máximo da 'função'? Tentei fazer a derivada mas não concluí nada,
acho que fiz errado. Num livro eu acho ter lido que o valor mais alto é
e^(1/e) mas já vi que com 2 o valor obtido é mais alto, acho. Um grande
abraço, Benjamin Hinrichs



   Caro Benjamin, tentei fazer o problema por derivadas. Veja a minha
solução:
   y=x^(1/x) - lny=lnx^(1/x)=(1/x)*lnx - d(lny)=d((1/x)*lnx)
   (1/y)*y'=(-x^(-2))*lnx+(1/x)*(1/x)
   (1/(x^1/x))*y'=(1/x^2)-(1/x^2)*lnx=(1/x^2)(1-lnx)
   y'=(1/x^2)(1-lnx)*(x^(1/x))=(x^((1-2x)/x))(1-lnx)
   Como sabemos, para y máximo, y'=0, para acharmos x:
   Da derivada acima determinada:
   (1-lnx)=0 - x=e - máx(y)=e^(1/e)
   Creio que o livro esteja correto. Confira também com as outras respostas
aqui da lista, ok?
   Um abraço!
Alexandre S. Gomes.



Get Your Private, Free E-mail from MSN Hotmail at http://www.hotmail.com





Re: Complexos - urgente

2000-05-16 Por tôpico José Paulo Carneiro

Nao entendi bem esta curva. nao serah a cos t+ i b sen t,
ou algo parecido?
JP

-Mensagem original-
De: Wellington Ribeiro de Assis [EMAIL PROTECTED]
Para: [EMAIL PROTECTED] [EMAIL PROTECTED]
Data: Segunda-feira, 15 de Maio de 2000 18:23
Assunto: Complexos - urgente


Prezados amigos da lista,

Estou com problemas com uma integral complexa e gostaria de poder 
contar o quanto antes com a ajuda de voces.


A questao eh a seguinte:

Calcular o indice de um caminho fechado w(A,0), onde 
A=a*sen(t)+b*cos(t), a,b0 e ab sem usar homologia. 
obs:  significa diferente.


definicao de w:
w(C,Zo)=[1/(2*pi*i)]*int(C)[dZ/(Z-Zo)], 
onde Zo nao pertence ao traco do caminho.

notacao:
'i'  eh a unidade imaginaria
'pi' eh o irracional
'int(C)' eh a integral no caminho C


De imediato podemos concluir que o valor desta integral eh 1, pois 
pelo teorema, como Zo=0, a substituicao da elipse pelo circulo de 
raio unitario nao altera o resultado pois os dois caminhos sao 
homologos.

Agradeco desde jah pelo tempo dispendido.
Abracos a todos,
Wellington


==
"Que os nossos esforcos desafiem as impossibilidades!
 Lembrai-vos de que as grandes proezas da humanidade
 Foram conquistadas daquilo que parecia impossivel".
Charles Chaplin

#ICQ: 13259380
Assis




Re: algorítmo da divisão

2000-05-11 Por tôpico José Paulo Carneiro

6k+5= 3(2k+1)+2.
A reciproca nao vale: basta ver que quando k eh par,
3k+2 eh par, enquanto 6k+5 eh impar.

-Mensagem original-
De: Marcelo Souza [EMAIL PROTECTED]
Para: [EMAIL PROTECTED] [EMAIL PROTECTED]
Data: Quinta-feira, 11 de Maio de 2000 14:12
Assunto: algorítmo da divisão


Olá pessoal
   Como faço para provar, utilizando o algorítmo da divisão, que todos
os números inteiros da forma 6K+5 são também da forma 3K+2, mas não vale a
recíproca?
Obrigado
Abraços
Marcelo

Get Your Private, Free E-mail from MSN Hotmail at http://www.hotmail.com





Re: [Fwd: Ajuda sobre teoria dos números...]

2000-05-06 Por tôpico José Paulo Carneiro




Eh interessante tambem o livro do 
S.C.Coutinho, cujo titulo eh
mais 
ou menos (cito de cabeca) Teoria dos Numeros e Criptografia RSA,
editado pela SBM.


-Mensagem original-De: 
Benedito Tadeu V. Freire [EMAIL PROTECTED]Para: [EMAIL PROTECTED] [EMAIL PROTECTED]Data: 
Sexta-feira, 5 de Maio de 2000 09:41Assunto: Re: [Fwd: Ajuda 
sobre teoria dos nmeros...]Caro Flvio, 
Segue uma bibliografia que talvez seja interessante voc 
acessar: a) Introduo  Teoria dos Nmeros, 
de Jos Plnio de Oliveira Santos -Rio de Janeiro - 
Coleo Matemtica Universitria- SBM b) An 
Introduction to the Theory of Numbers - Ivan M. Niven and 
Herbert S. Zuckerman . New York. John Willey  Sons. 1991 
Benedito Freire 
Flavio Borges Botelho wrote: 
Alexandre de Moura wrote: 
Preciso de 
ajuda, Podem me ajudar 
com a introduo e problemas sobre Teoria dos 
nmeros . . . [] desde 
j muito obrigado, Alexandre de Moura[EMAIL PROTECTED] 



Re: critica

2000-05-03 Por tôpico José Paulo Carneiro

So para acrescentar, Paulo.
Nao me lembro de ninguem ter dito que a pergunta
era "idiota ou imbecil". Em particular, eu falei justamente
o contrario: que problemas como esses sempre atarantaram os
matematicos, e nates deles, os filosofos, como Parmenides e 
Zenao de Eleia, por exemplo.
JP
 
-Mensagem original-
De: Paulo Santa Rita [EMAIL PROTECTED]
Para: [EMAIL PROTECTED] [EMAIL PROTECTED]
Data: Quarta-feira, 3 de Maio de 2000 11:30
Assunto: Re: critica


Ola Elon,
Tudo legal ?

Uma forma de responder a sua questao seria observar que que
0,999... pode ser entendido como 

9/10 + 9/100 + 9/1000 + ...

Esta "serie geometrica" e convergente e sua soma vale:

(9/10) / (1 - 1/10) = (9/10) / (9/10) = 1

Assim, "de fato", 0,999... = 1. Satisfaz ?

***

Eu nunca vi alguem pensar por postulados, axiomas e teoremas
... Eu sempre vi as pessoas justificarem suas percepcoes
internas e intuicoes com estas coisas ! Em "Prova e
Refutacao", de Lakatos, vemos que a famosa e sempre exigida
demonstracao nao passa de um estagio final de um longo e
laborioso processo mental dialectico,  totalmente
informalizavel e que, efetivamente, e o verdadeiro veiculo
de todas as descobertas cientificas que a humanidade ja fez.

Por outro lado, quem quer prova pra tudo deveria ler um
pouco de logica pois hoje sabemos que qualquer sistema
formal que use o minimo de principio aritmeticos  contera
proposicoes acerca dos objetos do sistema que sao totalmente
inatingiveis com os recursos de inferencia do mesmo
sistema(incompletude) ou tera contradicoes internas
(inconsistencia) - Teorema de Godel

***

Nao fique chateado se alguem, eventualmente, achou sua
pergunta algo idiota ou imbecil : a historia da ciencia
mostra que grandes descobertas so foram possiveis porque
alguns homens resolveram se fazer ( e responder ) perguntas
imbecis...

Quando alguem "sente" que aquilo que a comunidade acha
evidente nao e tao evidente assim, em geral, essa pessoa
esta percebendo estrutura digna de analise naquilo que os
outros acham irredutivel... isso pode ser muito bom !

Finalmente, a imensa maioria das pessoas da lista sao muito
legais, com grandes professores a nos orientarem.

Um abraco
Paulo Santa Rita
4,1013,03052000


On Fri, 21 Apr 2000 10:59:27 -0300
=?iso-8859-1?Q?Elon_Santos_Corr=EAa?= [EMAIL PROTECTED]
wrote:
Amigos,
 
a alguns dias enviei um e-mail para a lista intitulado "o
dia que nao acaba" e confesso que fiquei impressionado com
algumas respostas, estas respondiam como se a pergunta que
fiz (1 = 0,999...?) fosse uma ofensa a tudo aquilo que os
matematicos sabem, acreditam e aceitam.
A Matematica nao e' a verdade absoluta, nem o conjunto dos
numeros reais pode explicar tudo que ocorre no mundo
verdadeiro.
A essencia da questao foi perdida, prevaleceram os dogmas.
Nao estou defendendo aqui, que a resposta para a pergunta
que fiz seja sim ou nao, mas com certeza esta nao deveria
ser respondida atraves de respostas prontas, mas sim
questionada de forma logica e convincente.
E' esta atitude de alguns colegas da lista que por muitas
vezes afastam da matematica aqueles que por um ou outro
motivo, nao gostam ou nao estao tao preocupados com as
questoes puramente matematicas (nao e' o meu caso).

Sera' que Matematica e' pensar somente por axiomas,
postulados, teoremas, etc.?
 
Duas "coisas" sao iguais porque nao podemos provar (dentro
de regras limitadas) que elas sao diferentes? E se as
regras falharem?
 
De qualquer forma, agradeco a todas as respostas, Elon.


Don't E-Mail, ZipMail! http://www.zipmail.com/




Re: Problema de inteiros

2000-04-24 Por tôpico José Paulo Carneiro

Se nao forem primos entre si, eh falso. Como voce vai obter 5, que eh impar,
como uma soma de multiplos de 4 e 6?

-Mensagem original-
De: Marcos Eike Tinen dos Santos [EMAIL PROTECTED]
Para: [EMAIL PROTECTED] [EMAIL PROTECTED]
Data: Segunda-feira, 24 de Abril de 2000 11:06
Assunto: Re: Problema de inteiros


Um dos fatos importantes a ser considerado, é: Por que o problema nos impõe
a propriedade de que eles devem ser primos entre si. Será que foi por
acaso?
Eu penso que essa resposta é a metade do caminho para uma solução bem
formulada.

Ats,
Marcos Eike


- Original Message -
From: Ecass Dodebel [EMAIL PROTECTED]
To: [EMAIL PROTECTED]
Sent: Sábado, 22 de Abril de 2000 17:42
Subject: Problema de inteiros


 E ai, pessoal?

 Eu estava tentando resolver um dos problemas propostos na última Eureka!
e
 acabei chegando em uma parte que consigo seguir adiante mas é muito
 trabalhosa a minha prova, e não sei se está bem certa. Lá vai.

 1) Sejam x e y dois números primos entre si. Provar que podemos obter
 qualquer número somando múltiplos de x e de y.

 Solução.
 Queremos provar que para todo o x,y,n dados, podemos achar f e g de modo
que

 fx + gy = n  ( a soma de múltiplos de x e de y dão o n )

 Isola-se o f, ou o g... no caso isolei o f:

 f = (n - gy)/x

 Agora nos basta encontrar g de modo que x | n - gy. Para quem sabe um
 pouquinho de Teoria dos Números, eu acho que se variarmos o y num s.c.r.
 então o n - gy será um s.c.r. módulo x, e estaria provado. Mas vamos por
 partes:

 Suponhamos que

 n - g1y =/= n - g2y (mod x)'=/= incongruente
 g1y =/= g2y (mod x)  == afirmação similar a x não divide y(g1-g2)

 Como  mdc(x,y)=1 então

 g1 =/= g2 (mod x)

 Vale tambem que se g1 =/= g2 (mod x) então n - g1y =/= n - g2y (mod x).
 Agora escolhemos x números incongruentes módulo x (g1,...,gx), ou seja,
que
 nunca deixem o mesmo resto na divisão por x. E necessariamente:

 n - giy =/= n - gjy (mod x) para todo o i e j

 Ou seja, nesses x números (n-g1y,...,n-gxy), todos são incongruentes
módulo
 x, e como existem apenas x restos possíveis na divisão por x,
 necessariamente algum deles deixará resto zero na divisão por x, e
portanto
 haverá um g, tal que:

 f = (n - gy)/x será inteiro, e está provado o enunciado.

 2) Sejam x e y dois números primos entre si. Prove que existe um N, de
modo
 que para todo o n  N, podemos escolher múltiplos positivos de x e de y
que
 somados dão n. Nessas condições teremos que ter

 Solução.
 O problema pede para que mostremos que existem f e g positivos de modo
que,
 para n  N

 fx + gy = n  (lembrando que é todo mundo inteiro nesse e-mail)

 A minha idéia é a seguinte, claramente xy - yx = 0, e portanto para todo
o
a
 vale axy - ayx = 0, daí:

 fx + gy + axy - ayx = n
 (f + ay)x + (g - ax)y = n, para qualquer a que escolhermos

 Quero mostrar que existirá um a, a partir de um dado n, para que f + ay e
g
 - ax sejam ambos positivos.

 Conseguimos escolher a de modo que (f + ay)x - (g - ax)y = fx - gy + 2ayx
 esteja entre -yx e yx, basta mostrar que nesse intervalo teremos f+ay e
g-ax
 sempre positivos.

 Tanto f+ay quanto g-ax podem ficar entre [ n-xy ; n+xy ], ou seja basta
que
 n-xy0 e portanto que n  xy. Logo para N = xy vale o enunciado.


 Obrigado para quem leu! E tem algum erro?
 Valeu...
 
 Get Your Private, Free E-mail from MSN Hotmail at http://www.hotmail.com






Re: Problema de inteiros

2000-04-23 Por tôpico José Paulo Carneiro

O Teorema de Bezout (por alguns chamado de Teorema Fundamental da
Teoria dos Numeros) diz que dados dois inteiros a e b, eh possivel (usando o
algoritmo de Euclides, por exemplo) encontrar dois inteiros r e s tais que
ra+sb=m.d.c(a,b). [Veja RPM no 37, p.27: "Dispositivo pratico para expressar
o mdc de dois numeros como combinacao linear deles"].
Em particular, se a e b e forem primos entre si, pode-se achar r e s tais
que
ra+sb=1 e, portanto:  (r n)a+ (sn)b = n, onde n eh um numero qualquer dado.


PS: Eu ja tinha escrito este e-mail quando vi que o Marcos Paulo falou a
mesma coisa. That's my boy!

-Mensagem original-
De: Ecass Dodebel [EMAIL PROTECTED]
Para: [EMAIL PROTECTED] [EMAIL PROTECTED]
Data: Sábado, 22 de Abril de 2000 18:05
Assunto: Problema de inteiros


E ai, pessoal?

Eu estava tentando resolver um dos problemas propostos na última Eureka! e
acabei chegando em uma parte que consigo seguir adiante mas é muito
trabalhosa a minha prova, e não sei se está bem certa. Lá vai.

1) Sejam x e y dois números primos entre si. Provar que podemos obter
qualquer número somando múltiplos de x e de y.

Solução.
Queremos provar que para todo o x,y,n dados, podemos achar f e g de modo
que

fx + gy = n  ( a soma de múltiplos de x e de y dão o n )

Isola-se o f, ou o g... no caso isolei o f:

f = (n - gy)/x

Agora nos basta encontrar g de modo que x | n - gy. Para quem sabe um
pouquinho de Teoria dos Números, eu acho que se variarmos o y num s.c.r.
então o n - gy será um s.c.r. módulo x, e estaria provado. Mas vamos por
partes:

Suponhamos que

n - g1y =/= n - g2y (mod x)'=/= incongruente
g1y =/= g2y (mod x)  == afirmação similar a x não divide y(g1-g2)

Como  mdc(x,y)=1 então

g1 =/= g2 (mod x)

Vale tambem que se g1 =/= g2 (mod x) então n - g1y =/= n - g2y (mod x).
Agora escolhemos x números incongruentes módulo x (g1,...,gx), ou seja, que
nunca deixem o mesmo resto na divisão por x. E necessariamente:

n - giy =/= n - gjy (mod x) para todo o i e j

Ou seja, nesses x números (n-g1y,...,n-gxy), todos são incongruentes módulo
x, e como existem apenas x restos possíveis na divisão por x,
necessariamente algum deles deixará resto zero na divisão por x, e portanto
haverá um g, tal que:

f = (n - gy)/x será inteiro, e está provado o enunciado.

2) Sejam x e y dois números primos entre si. Prove que existe um N, de modo
que para todo o n  N, podemos escolher múltiplos positivos de x e de y que
somados dão n. Nessas condições teremos que ter

Solução.
O problema pede para que mostremos que existem f e g positivos de modo que,
para n  N

fx + gy = n  (lembrando que é todo mundo inteiro nesse e-mail)

A minha idéia é a seguinte, claramente xy - yx = 0, e portanto para todo o
a
vale axy - ayx = 0, daí:

fx + gy + axy - ayx = n
(f + ay)x + (g - ax)y = n, para qualquer a que escolhermos

Quero mostrar que existirá um a, a partir de um dado n, para que f + ay e g
- ax sejam ambos positivos.

Conseguimos escolher a de modo que (f + ay)x - (g - ax)y = fx - gy + 2ayx
esteja entre -yx e yx, basta mostrar que nesse intervalo teremos f+ay e
g-ax
sempre positivos.

Tanto f+ay quanto g-ax podem ficar entre [ n-xy ; n+xy ], ou seja basta que
n-xy0 e portanto que n  xy. Logo para N = xy vale o enunciado.


Obrigado para quem leu! E tem algum erro?
Valeu...

Get Your Private, Free E-mail from MSN Hotmail at http://www.hotmail.com






Re: O dia que nao acaba

2000-04-20 Por tôpico José Paulo Carneiro

Caro Flavio:
Voce pode imaginar que este tipo de discussao atarantou os
matematicos durante muitos anos. Para evitar que a Matematica
estagnasse ou que descambasse para discussoes do tipo: "eu
acho que 2+2=5 porque sou budista", os matematicos, desde a
segunda metade do seculo XIX (culminando talvez com Hilbert,
na virada para o seculo XX), desenvolveram uma atitude mais
prgamatica, convencionando um sistema mais modesto de "certezas"
e de metodos aceitos por todos (os matematicos), a partir dos quais
eh possivel deduzir tudo que um matematico precisa para trabalhar
e aplicar a matematica ao mundo fisico. Tal sistema consta de conceitos
primitivos, axiomas, e regras aceitas de Logica, e a partir dahi fluem os
teoremas conhecidos. Dentro deste sistema, nao
ha lugar para "numeros infinitesimais" ou para consideracoes esotericas
sobre o infinito. E dentro de tal sistema, como ja disse em outro e-mail,
nao
ha a menor duvida de que 0,999...=1. Agora, ninguem pode impedir ninguem
de atribuir outros significados a esses termos e fazer suas proprias
consideracoes "filosoficas". Mas isto nao eh considerado propriamente
matematica pelos matematicos.
JP

Post Scriptum: Sei muito bem que este sistema que mencionei apresenta certas
dificuldades, e que ha uma teoria (chamada "Non-standard analysis") que
tenta
resgatar conceitos tipo "numero infinitesimal", mas para analisar isto, so
escrevendo 900 paginas.
JP





-Mensagem original-
De: Flavio Borges Botelho [EMAIL PROTECTED]
Para: [EMAIL PROTECTED] [EMAIL PROTECTED]
Data: Quarta-feira, 19 de Abril de 2000 23:51
Assunto: Re: O dia que nao acaba


Ralph Costa Teixeira wrote:

 Assim, eu digo: acostume-se com a ideia de que 1,...=2,
exatamente,
 mesmo que a sua intuicao inicial seja capaz de jurar que 1,...2
 (sua intuicao estah comparando digitos, o que soh nao funciona no caso
 de dizimas com noves).

Espero não deixar os defensores bravos, mas não pode ser possível que a
linha
dos reais está furada? Faltando certos números 'infinitesimais' que seriam
o caso
da diferença entre 1,9 e 2?
Até hoje uma definição matemática para infinito nós escapa, e quem sabe
seja
impossível  encontrá-la... Em geral o que fazermos é levarmos aos limites,
e esse
caso parece ser um caso onde esse limite de infinito parece estar
impregnado...

lim (n - infinito) 2 - 10^(-n) = 1,9

O problema que quando lidamos com 1,... como um número real, a
parte
infinitesimal acaba desaparecendo e chegamos a conclusão que é igual a 2.


 Abraço,
 Ralph

Abração,
Flávio






Re: Congruências e Raizes n-ésimas da unidade

2000-03-21 Por tôpico José Paulo Carneiro

Ola Marcos:
Bem lembrado!
O livro alias eh do Wagner. So o apendice eh meu. E eh no apendice
que estah o que voce fala.
Nao vou repetir aqui, porque eh bom voces terem o livro, para lerem
outras coisas interessantes.
JP

-Mensagem original-
De: Marcos Paulo [EMAIL PROTECTED]
Para: [EMAIL PROTECTED] [EMAIL PROTECTED]
Data: Terça-feira, 21 de Março de 2000 02:26
Assunto: RES: Congruências e Raizes n-ésimas da unidade



No livro "Construções Geométricas"  - Eduardo Wagner e J.P.Q. Carneiro tem
um apendice q trata do assunto que vc pediu!!
Um exercício interessante q é apresentado nesse apendice é a solução da
equação binômia  x^5 -1=0 que o Próprio JP poderia expor aki!
esse assunto é muito interessante!!!
[]'s M.P.
-Mensagem original-
De: [EMAIL PROTECTED] [mailto:[EMAIL PROTECTED]]Em
nome de [EMAIL PROTECTED]
Enviada em: segunda-feira, 20 de março de 2000 10:59
Para: [EMAIL PROTECTED]
Assunto: Congruências e Raizes n-ésimas da unidade


Oi gente...

Li num livro de álgebra (pelo menos foi o que entendi, se entendi errado
esclareçam!)que Gauss (Salve!!), após ter desenvolvido sua teoria
de "congruências" usou seus resultados para atacar o problema das raizes n-
ésimas da unidade, gerando ao final os métodos para construções com régua
e compasso de polígonos regulares.
ALLguém saberia me dar mais informações sobre essa relação? Ou pelo menos
citar uma fonte para consulta?

Grato e,
Saudações(Tricolores, claro!!!)

Alexandre Vellasquez





Re: Congruências e Raizes n-ésimas da unidade

2000-03-21 Por tôpico José Paulo Carneiro

Por exemplo: as n raizes n-esimas de 1 formam, no plano complexo,
os vertices de um poligono regular de n lados inscrito no circulo unitario.
Por outro lado, a "primeira" raiz u depois do proprio 1, isto eh cis(2pi/n),
eh tal que u^r=u^s se e so se r eh congruo de s modulo n.



-Mensagem original-
De: [EMAIL PROTECTED] [EMAIL PROTECTED]
Para: [EMAIL PROTECTED] [EMAIL PROTECTED]
Data: Segunda-feira, 20 de Março de 2000 14:14
Assunto: Congruências e Raizes n-ésimas da unidade


Oi gente...

Li num livro de álgebra (pelo menos foi o que entendi, se entendi errado
esclareçam!)que Gauss (Salve!!), após ter desenvolvido sua teoria
de "congruências" usou seus resultados para atacar o problema das raizes n-
ésimas da unidade, gerando ao final os métodos para construções com régua
e compasso de polígonos regulares.
ALLguém saberia me dar mais informações sobre essa relação? Ou pelo menos
citar uma fonte para consulta?

Grato e,
Saudações(Tricolores, claro!!!)

Alexandre Vellasquez




Re: sistema

2000-03-21 Por tôpico José Paulo Carneiro


-Mensagem original-
De: Ralph Costa Teixeira [EMAIL PROTECTED]
Para: [EMAIL PROTECTED] [EMAIL PROTECTED]
Data: Terça-feira, 21 de Março de 2000 22:23
Assunto: Re: sistema


 Oi, Elon.

 Argh, estou sem tempo para comentar isso, então cuidado: grande
possibilidade de eu falar besteira aí embaixo! Vou "xutar" umas coisas,
por favor verifiquem.

 Basicamente, você quer x^y=y^x com yx0, certo? Afinal, depois que
você encontrá-los, basta tomar a=y/x e a temos uma solução da sua
equação.

 Parte deste problema já foi discutido aqui, disfarçado... Tinha um
thread sobre x^x^x^x^x..., cujo artigo mais informativo é o do Gugu:

http://www.mat.puc-rio.br/~nicolau/olimp/obm-rj.1999/msg00169.html

 Siga o thread todo se você não lembra. O que o Gugu disse (e que eu não
sei se é fácil de provar) é que y=x^x^x^x... (definido de cima para
baixo, como no artigo do Gugu) existe quando e^(-e)  x  e^(1/e); neste
caso, x^y = y^x e y  x... eu acho.

 Acredito que para x fora desse intervalo não há solução com y!=x...

 Acho que há um artigo numa das revistas da AMS que trata disso... e
acho que há algo assim na RPM também...


E eu ja dei esta referencia: eh um artigo do Wagner. (JP)


 Finalmente, acho que dá para mostrar a existência da solução para f(y)
= x^y-y^x = 0 para um x fixo naquele intervalo usando um pouco de
cálculo... talvez não para o intervalo do Gugu, mas para intervalos
menores.

 Finalmente, a única solução inteira, acredito ser 2^4=4^2, se é isso
que você quer.

 Argh, tantos achismos, mas eu tenho que ir embora para pegar minha
carona... :)

 Abraço,
 Ralph


 Elon Santos Corrêa wrote:

 Olá pessoal da lista,

 gostaria de ver soluções para o sistema de equações:

 x^y = y^x
 e
 y = ax

   com a  0 e diferente de 1.

 Obs.: o símbolo " ^ " significa elevado ao expoente ...

 Antecipadamente agradeço as respostas, Elon.




Re: interessante

2000-03-21 Por tôpico José Paulo Carneiro

$@ *^$~#,
isto eh eu recebi.

-Mensagem original-
De: Eduardo Wagner [EMAIL PROTECTED]
Para: [EMAIL PROTECTED] [EMAIL PROTECTED]
Data: Terça-feira, 21 de Março de 2000 21:56
Assunto: Re: interessante


 On Tue, 21 Mar 2000, José Fabrício Maia wrote:

  Colegas gostaria de saber como se mostra que raiz quadrada de 2
elevado a raiz quadrada de 2 é irracional.

 Aqui vai um problema relacionado que não é tão difícil (talvez até seja
o problema que inspirou a questão do José, não sei...), a ser resolvido
sem uso de nenhum teorema pesado:

Problema:
 Prove que existem números irracionais a e b tais que a^b é racional.

Solução curta (a ser completada):
 "Se a afirmativa do José Maia não for verdadeira $^(#^$%%CARRIER OUT
e está provado.

 Por outro lado, se a afirmativa do José Maia for verdadeira...
%$%Q%#%U$E A$Z$A^$R D%E# N$O###V^*$$O e então basta notar que
#J%^US%^$T(^O*^ $AG^%O#$$@R##A#? ... e acabou."

 Abraço,
 Ralph

EPA: voces tambem receberam a brilhante porem ininteligivel mensagem do
Ralph?






Re: matemática no enem

2000-03-08 Por tôpico José Paulo Carneiro

Toda definicao eh feita em funcao de termos considerados previamente
conhecidos.
Por exemplo, se defino triangulo equilatero como aquele que tem os tres
lados com mesmo comprimento, eh porque estou supondo que meu
interlocutor sabe o que eh triangulo, o que eh lado, o que eh ter o mesmo
comprimento, etc. Se ele nao sabe o que eh triangulo, definimos entao
triangulo em termos de pontos, segmentos, etc. E assim por diante.

Como esta cadeia de definicoes nao pode continuar indefinidamente,
somos obrigados a admitir (em qualquer parte da Matematica, nao so
na geometria) alguns conceitos que nao se definem em termos de outros
mais conhecidos. Estes sao os chamados "conceitos primitivos" (nao tem
nada a ver com "intuitivo"). A escolha de quais conceitos sao primitivos ou
sao definidos depende de quem estah organizando a teoria. Mas no caso
da Geometria, todos consideram ponto e reta como conceito primitivo (exceto
em certas abordagens algebricas da Geometria).

Voce dirah: Mas se ponto e reta nao sao definidos, como vamos saber o que
sao, como vamos trabalhar com eles em Matematica?  A resposta a isto estah
nos axiomas que acompanham cada conceito primitivo (por exemplo: cada dois
pontos estao contidos em uma unica reta). Os axiomas estao para os teoremas
assim como os conceitos primitivos estao para as definicoes. So se pode usar
os
conceitos primitivos de acordo com o que estah nos axiomas.
JP






-Mensagem original-
De: Marcos Eike Tinen dos Santos [EMAIL PROTECTED]
Para: [EMAIL PROTECTED] [EMAIL PROTECTED]
Data: Quarta-feira, 8 de Março de 2000 01:41
Assunto: Re: matemática no enem


Prezado JP. fico grato por responder a minha mensagem, porém não há
possibilidades de vc enunciar a definição de ponto e reta? Pois, não tenho
acesso a essas revistas.

Agradeceria muito.

Marcos Eike Tinen dos Santos





Re: raizes

2000-03-06 Por tôpico José Paulo Carneiro

Calcular a raiz n-esima de M eh o mesmo que resolver a equacao polinomial
x^n=M. Voce pode usar, por exemplo, o metodo de Newton para equacoes
polinomiais. Veja por exemplo o livro que ja indiquei aqui:
Resolucao de Equacoes Algebricas, de J.P.Carneiro, editado pela Universidade
Santa Ursula. Veja tambem o artigo na RPM 40, p.31.
JP

-Mensagem original-
De: Marcelo Souza [EMAIL PROTECTED]
Para: [EMAIL PROTECTED] [EMAIL PROTECTED]
Data: Domingo, 5 de Março de 2000 18:06
Assunto: raizes


Ola pessoal
Alguem poderia me indicar um bom livro que ensinasse maneiras de se
calcular
raizes quadradas, cubicas, exatas e nao-exatas?
Muito obrigado antecipadamente
Abraços
Marcelo
__
Get Your Private, Free Email at http://www.hotmail.com




Re: matemática no enem

2000-03-06 Por tôpico José Paulo Carneiro




Participo plenamente de suas 
apreensoes. Eh o velho movimento pendular
da 
historia das ideias.
Jose Paulo Carneiro

-Mensagem original-De: 
Jos Fabrcio Maia [EMAIL PROTECTED]Para: 
discusso de problemas [EMAIL PROTECTED]Data: 
Segunda-feira, 6 de Maro de 2000 14:19Assunto: 
matemtica no enem

Caros amigos da lista, no sei se o assunto  adequado, mas 
no vejo prejuzo nenhum para a lista. Gostaria de ter a 
opinio de vocs. 
Vou tentar ser breve: O exame nacional 
prope situaes prticas, contextualizadas, 
exigindo do aluno, raciocnio e no simplesmente 
memorizao de frmulas. No vejo problema 
at a. No entanto muitas questes exibidas nas provas 
anteriores, embora bem boladas, no abrange toda a matemtica 
que se ensina em sala de aula. Continuando nesse ritmo, ento, existe 
a possibilidade de se excluir naturalmente determinados contedos do 
vestibular e consequentemente dos currculos dessa nova 
gerao. Ser que daqui a alguns anos esse conhecimento 
elaborado, adquirido atravs de grandes mestres, textos de livros e 
revistas no vo se tornar inadequados as novas propostas de 
ensino por no ser to prtico? Tal 
conhecimento no ficar mais elitizado ou mesmo esquecido? O 
conhecimento elaborado e o conhecimento cotidiano na minha opinio 
devem caminhar juntos. Mas, a partir do momento que se exclui uma 
porcentagem to alta de contedos matemticos, parece 
at que apenas uma pequena parte do que se ensina tem utilidade 
prtica. Todo conhecimento tem sua importncia em nosso 
crescimento. Mas do jeito que as coisas ocorrem, parece at que foi 
um desperdcio aprender tanto.
[EMAIL PROTECTED]
Grato!


Re: Entendendo à Aplicação dos números complexos na geometria

2000-03-01 Por tôpico José Paulo Carneiro

Indico o meu livro:
Resolucao de Equacoes Algebricas, 1998, que pode ser comprado na
Universidade Santa Ursula, predio VI, 12o andar.  O primeiro capitulo eh
dedicado aos
numeros complexos, e a abordagem eh geometrica.
Jose Paulo Carneiro

-Mensagem original-
De: Marcos Eike Tinen dos Santos @ ITA @ [EMAIL PROTECTED]
Para: [EMAIL PROTECTED] [EMAIL PROTECTED]
Data: Domingo, 6 de Fevereiro de 2000 01:13
Assunto: Entendendo à Aplicação dos números complexos na geometria



Oi,

Quem poderia me explicar o uso desta teoria? Li o artigo no Eureka, entendi
a parte que eles mostram as raízes complexas dispostas na circunferência
tendo n lados. Porém, não entendi à aplicação nos exercícios.
Os números complexos se não estou enganado pode ser usado como vetores,
aguém poderia me explicar?
Caso seja possível por favor uma explicação completa. Será que encontro
aqui
no Brasil totalmente ligado ao assunto?

Outro Problema, Alguém conhece a  função pi(x)? Quer me explicar?
Tem um exercício, porém não entendi direito.

O exercício pede para Provar que há infinitos número primos congruentes a 1
mod 4.



Muito Obrigado!

Marcos Eike Tinen dos Santos






Re: Fibonacci

1999-12-24 Por tôpico José Paulo Carneiro

Caro Eric:
(Veja abaixo)

-Mensagem original-
De: Eric Campos [EMAIL PROTECTED]
Para: Lista da OBM [EMAIL PROTECTED]
Data: Sexta-feira, 24 de Dezembro de 1999 00:36
Assunto: Fibonacci


O seguinte problema foi proposto por Jose Paulo Carneiro

As duas sequencias:
1, 1, 2, 3, 5, 8, 13, ...
e
1, 3, 4, 7, 11, ...

Sao ambas tais que cada termo eh a soma dos dois anteriores.
Observacoes: a segunda sequencia eh chamada "sequencia de Lucas".

Mostre que:

dado um inteiro positivo arbitrario m, existe sempre algum termo da
primeira sequencia que eh multiplo de m.

Imaginei uma solucao assim:

seja f(i) o i-esimo termo da sucessao de Fibonacci 1, 1, 2, 3, 5, 8, 13,
..., em que cada termo eh a soma dos dois anteriores. Seja m um inteiro
generico. Considere a funcao g(n) = resto de f(n) quando dividido por m,
com
g(n) nao negativo. Eh claro que g eh uma sucessao, pois g esta definida no
conjunto dos naturais. Como ha no maximo m distintos valores possiveis para
g(n), ha no maximo m^2 distintos possiveis pares de inteiros consecutivos
na
sucessao g. Portanto existem inteiros positivos i,k para os quais
(g(i),g(i+1)) = (g(i+k),g(i+k+1)), isto eh, existem dois pares de numeros
de
Fibonacci consecutivos que tem o mesmo resto (positivo) na divisao por m.
Nesse caso como g(i) + g(i+1) = g(i+k) + g(i+k+1) temos em particular que
g(i+2) = g(i+k+2),

*
Ateh aqui, eu estava concordando plenamente com voce.
*

e de modo mais geral g(n) = g(n + k) para cada n inteiro
positivo.

*
Agora, este passo nao estah claro para mim. So sabiamos isto para n
particulares, da forma i+2.
Se fizermos "experiencias", com m=2,3,4,5,6,..., vamos observar que
o par de restos que repete primeiro eh sempre o (1,1). Isto sugere o
seguinte caminho que proponho:
Seja i* o menor i para o qual ocorre a igualdade (g(i),g(i+1)) =
(g(i+k),g(i+k+1)).
Se i*1, entao, como g(i*+1)=g(i*)+g(i*-1)  e g(i*+k+1)=g(i*+k)+g(i*+k-1),
conclui-se que g(i*-1)=g(i*+1)-g(i*)=g(i*+k-1)=g(i*+k+1)-g(i*+k).
Mas entao, o par (g(i*+k-1), g(i*+k)) ja repetiria o apr (g(i*-1), g(i*)), o
que eh
absurdo, pois contraria a definicao de i*. Logo, i*=1.
Agora sim: temos: (g(1), g(2))=(1,1)=(g(k+1), g(k+2)) . Logo:
g(k)=g(k+2)-g(k+1)=1-1=0, e portanto g(k) eh multiplo de m.

Jose Paulo
**

logo 1 = g(1) = g(1 + k) e 1 = g(2) = g(2 + k), donde g(k) = g(2 +
k) - g(1 + k) = 0, isto eh, m divide f(k).

Eric.


A partir daquele ponto, sugiro o seguinte:



Re: taxa de importação

1999-12-17 Por tôpico José Paulo Carneiro




Ja comprei livros na Amazon mais de uma duzia de 
vezes, e nunca me cobraram
taxa de importacao. Livros estao isentos, mas CDs 
nao.
JP

-Mensagem original-De: 
Antonio Nascimento Filho [EMAIL PROTECTED]Para: 
Lista de Matemtica [EMAIL PROTECTED]Data: 
Sbado, 18 de Dezembro de 1999 02:20Assunto: taxa de 
importao
Bruno Leite Wrote:

No h exemplares desse livro 
na USP nem na UNICAMP. Na Amazon.com tem oivro, mas  meio 
caro(33 dlares x 1,90 + taxa de importao 60% etc) 
edeve demorar a chegar, se encomendado.

 Sei q no tem nada a 
ver com matemtica, mas como no tenho a quem perguntar, vai 
aqui mesmo..
Tenho ouvido falar dessa tal taxa de 
importao, e no fao idia de como ela 
se aplica. J vi gente reclamando q pagou isso em cd's, outros em 
livros, mas eu pedi um livro na Amazon a menos de um ms atrs 
(24 dlares com a taxa de remessa) e no me cobraram nada de 
taxa de importao!
 Ser que 
algum pode me explicar isso ou indicar um lugar onde se possa obter 
uma informao bem clara a respeito??


 Abraos
 
Antonio


PS: Peo desculpas ao Nicolau por essa 
msg no matemtica, mas esse assunto tem me deixado meio 
confuso e creio q seja do interesse de muitos na 
lista.